Thematic Test - 7 With Answers.pdf

  • Uploaded by: rahul rajvardhan
  • 0
  • 0
  • December 2019
  • PDF

This document was uploaded by user and they confirmed that they have the permission to share it. If you are author or own the copyright of this book, please report to us by using this DMCA report form. Report DMCA


Overview

Download & View Thematic Test - 7 With Answers.pdf as PDF for free.

More details

  • Words: 25,337
  • Pages: 66
Test is part of Rau’s IAS Test series for Preliminary Exam 201 9

THEMATIC + CURRENT AFFAIRS

Test Code

GENERAL STUDIES (PAPER –I) THEMATIC TEST–VII

TC19E1007

TOPIC: AIH + Government Policies + Current Affairs Time Allowed: 2 Hours

Maximum Marks : 200

(i) (ii)

(iii)

This test is part of Rau’s IAS Test series for Preliminary Exam 201 9

Q.1)

मौर्य काल के सन्दर्य में निम्ननलखित र्ु ग्मों पर निचार

Q.4)

कीनिए: 1.

समाहताय

:

रािकीर् र्ोंडार

कमषागार गृ ह

का

और प्रमुि

सोंरक्षक 2.

सनिधाता

:

मूल्ाोंकि और सोंग्रह के

उपर्ुयक्त र्ु ग्मों में से कौि-सा/से सही सुमेनलत है/हैं? केिल 1

(b)

केिल 2

(c)

1 और 2 दमिमों

(d)

ि तम 1, ि ही 2 Q.5)

Q.2)

गु प्त काल के सोंदर्य में निम्ननलखित कथिमों में से कौिर्ास िे पदे के नलए ‘र्िनिका’ शब्द का उपर्मग नकर्ा है , िम र्ू िाि से सोंपकय का सुझाि दे ता है । 2.

गु प्त काल में पानिनि और पतोंिनल के कार्ों पर आधाररत सोंस्कृत व्याकरि का निकास दे िा गर्ा था।

3.

सोंस्कृत गु प्तमों की दरबारी र्ाषा थी।

4.

कुमारगुप्त द्वारा नलिी गई ‘अनर्ज्ञािशाकुन्तलम’, रािा दु ष्योंत और शकुोंतला की प्रेमकथा से सोंबोंनधत है ।

नसमुक सातिाहि िों श का सोंस्थापक था।

2.

सातिाहिमों की रािधािी प्रनतष्ठाि थी।

(a)

केिल 1

(b)

केिल 2

(c)

1 और 2 दमिमों

(d)

ि तम 1, ि ही 2

निम्ननलखित में से नकसे नििर्िगर दरबार में एक रािदू त के रूप में निर्ु क्त नकर्ा गर्ा था?

से सही हैं ? 1.

1.

उपर्ुयक्त कथिमों में से कौि-सा/से सही है /हैं ?

नलए सिोच्च अनधकारी

(a)

निम्ननलखित कथिमों पर निचार कीनिए:

Q.6)

(a)

माको पमलम (Marco Polo)

(b)

निकमलम कौोंटी (Nicolo Conti)

(c)

दु आते बारबमसा (Duarte Barbosa)

(d)

अब्दु र रज़्ज़ाक (Abdur Razzaq)

निम्ननलखित कथिमों पर निचार कीनिए: 1.

निष्पानदत बडे कार्ों कम ‘म्यूराल नचत्मों’

िीचे नदए गए कूट का प्रर्मग कर सही उत्तर चु निए:

Q.3)

(a)

केिल 1 और 2

(b)

केिल 3 और 4

(c)

केिल 1, 2 और 3

(d)

1, 2, 3 और 4

महािीर िे निम्ननलखित में से नकस र्ाषा/बमली में उपदे श नदए थे? (a)

मगधी

(b)

अधय-मगधी

(c)

शौरसेिी

(d)

अपभ्रोंश

RAUSIAS-TC19E1007

बडे र्ििमों की दीिारमों पर नचत्मों के रूप में

(Mural Paintings) के रूप में िािा िाता है । 2.

ले पाक्षी नचत् मौनलक रों गमों से र्रे हुए थे ।

उपर्ुयक्त कथिमों में से कौि-सा/से सही है /हैं ? (a)

केिल 1

(b)

केिल 2

(c)

1 और 2 दमिमों

(d)

ि तम 1, ि ही 2

2

Q.1)

Consider the following pairs regarding the Mauryan period: 1.

2.

Samaharta

Sannidhata

:

:

Q.4)

1.

The chief custodian of the state treasury and storehouse.

Q.2)

1 only

(b)

2 only

(c)

Both 1 and 2

(d)

Neither 1 nor 2

2.

1.

2.

Which of the statements given above is/are correct?

Q.5)

(a)

1 only

(b)

2 only

(c)

Both 1 and 2

(d)

Neither 1 nor 2

Who among the following was deputed to the court of Vijayanagar as an

Bhasa uses the term yavanika for the curtain, which suggests Greek contact.

ambassador? (a)

Marco Polo

The Gupta period saw the development of Sanskrit grammar based on the work of Panini and Patanjali.

(b)

Nicolo Conti

(c)

Duarte Barbosa

(d)

Abdur Razzaq

3.

Sanskrit was the court language of the Guptas.

4.

Abhijnanasshakuntalam, written by Kumargupta, relates to the love story of king Dushyanta and Shakuntalam.

Q.6)

Consider the following statements: 1.

(a)

1 and 2 only

(b)

3 and 4 only

(c)

1, 2 and 3 only

(d)

1, 2, 3 and 4

Mahavira preached in which of the following languages/dialects? (a)

Magadhi

(b)

Ardha – Magadhi

(c)

Suraseni

(d)

Apabhramsa

RAUSIAS-TC19E1007

Large works executed in the form of paintings on the walls of large structures are known as Mural

Select the correct answer using the code given below:

Q.3)

The capital of Satvahanas was Pratishthana.

The highest officer in charge of assessment and collection.

Which of the following statements are correct regarding the Gupta period?

Simuka was the founder of the Satvahana dynasty.

Which of the pairs given above is/are correctly matched? (a)

Consider the following statements:

paintings. 2.

Lepakshi paintings were full of primary colours.

Which of the statements given above is/are correct? (a)

1 only

(b)

2 only

(c)

Both 1 and 2

(d)

Neither 1 nor 2

3

Q.7)

अशमक

के

निम्ननलखित

अनर्ले ि/अनर्ले िमों में

में

से

प्रानर्क

नकस/नकि

िाम

Q.9)

नसोंधु घाटी सभ्यता के सन्दर्य में निम्ननलखित कथिमों पर निचार कीनिए:

'दे िािाम

नपर्दसी' के अनतररक्त ‘अशमक’ िाम का उल्लेि

1.

इस सभ्यता के लमग िृ क्षमों की पूिा करते थे।

2.

इसमें पशु ओों की पूिा र्ी की िाती थी तथा

नकर्ा गर्ा है ? 1.

मस्की

2.

गु िरय

3.

निट् टूर

4.

उदर्गमलम

उिमें से कई कम मुहरमों पर प्रदनशय त नकर्ा गर्ा है । इिमें एक सीोंग िाला पशु ‘गैं डा’ सिाय नधक महत्वपूिय है । 3.

निर्ेदि िहीों था। उपर्ुयक्त कथिमों में से कौि-सा/से सही है /हैं ?

िीचे नदए गए कूट का प्रर्मग कर सही उत्तर चु निए:

Q.8)

(a)

केिल 1 और 2

(b)

केिल 2 और 3

(c)

केिल 4

(d)

1, 2, 3 और 4

Q.10)

सातिाहिमों की आनधकाररक र्ाषा ‘प्राकृत’ थी।

2.

'गाथा सप्तसती' (एक प्राकृत ग्रन्थ) का श्रेर्

3.

(a)

केिल 1

(b)

केिल 2

(c)

केिल 1 और 3

(d)

केिल 1 और 2

पाण्डु और िरुि निम्ननलखित में से नकस िदी की सहार्क िनदर्ााँ हैं?

निम्ननलखित कथिमों पर निचार कीनिए: 1.

इस सभ्यता में नकसी प्रकार का सामानिक

Q.11)

(a)

र्मुिा

(b)

गमदािरी

(c)

कृष्णा

(d)

उपर्ुय क्त्त्त में से कमई िहीों

निम्ननलखित में से कौि-सी प्रनसद्ध मूनतय र्ााँ मौर् र् काल

सातिाहि सम्राट 'हाल' कम नदर्ा िाता है ।

से सम्बों नधत हैं?

सातिाहिमों िे ब्राह्मिमों तथा बौद्ध नर्क्षुओों कम

1.

सुल्ताि गों ि बु द्ध

2.

दीदार गों ि र्नक्षिी

3.

परिम र्क्ष

4.

धौली में रॉक-कट हाथी

कर-मुक्त गााँ ि दे िे की प्रथा आरम्भ की थी। उपर्ुयक्त कथिमों में से कौि-सा/से सही है /हैं ? (a)

केिल 3

(b)

केिल 1 और 2

(c) (d)

िीचे नदए गए कूट का प्रर्मग कर सही उत्तर चु निए: (a)

केिल 1, 2 और 4

(b)

केिल 1, 3 और 4

केिल 2 और 3

(c)

केिल 1, 2 और 3

1, 2 और 3

(d)

केिल 2, 3 और 4

RAUSIAS-TC19E1007

4

Q.7)

In which of the following Ashokan edicts the name "Ashoka", in addition to usual

Maski

2.

Gujarra

3.

Nittur

4.

Udaigolam

Consider

2.

The

people

this

civilization

the seals. The most important of them is the one-horned animal ‘Unicorn’. 3.

There was no social differentiation in this civilization.

Which of the statements given above

1 and 2 only

(b)

2 and 3 only

(a)

1 only

(c)

4 only

(b)

2 only

(d)

1, 2, 3 and 4

(c)

1 and 3 only

(d)

1 and 2 only

is/are correct?

Consider the following statements: official

language

of

Q.10) Pandu and Varuna are the tributaries of the

Satavahans was Prakrit. 2.

of

Animals were also worshipped and

(a)

The

statements

many of them are represented on

given below:

1.

following

worshipped trees.

Select the correct answer using the code

Q.8)

the

regarding the Indus Valley Civilization: 1.

"Devanam Piyadasi", is mentioned? 1.

Q.9)

‘Gatha Saptasati’ (a Prakrit text) is attributed to the Satavahana king

which of the following rivers? (a)

Yamuna river

(b)

Godavari river

(c)

Krishna river

(d)

None of the above

‘Hala’. 3.

The

Satavahans

practice

of

villages

to

started

granting Brahmans

the

tax-free and

Q.11) Which

of

sculptures

the belong

following to

the

famous Mauryan

period? 1.

Sultan Ganj Buddha

2.

Didar Ganj Yakshini

Which of the statements given above

3.

Parkham Yaksha

is/are correct?

4.

Rock-cut elephant at Dhauli

Buddhist monks.

Select the correct answer using the code

(a)

3 only

(b)

1 and 2 only

(a)

1, 2 and 4 only

(c)

2 and 3 only

(b)

1, 3 and 4 only

(d)

1, 2 and 3

(c)

1, 2 and 3 only

(d)

2, 3 and 4 only

RAUSIAS-TC19E1007

given below:

5

Q.12)

Q.13)

निम्ननलखित र्ु ग्मों में से कौि-सा सही सुमेनलत िहीों है ? (a)

न्यार्: गौतम

(b)

र्मग: पतों िनल

(c)

साों ख्य: बद्रार्ि

(d)

पूिय मीमाों सा: िै नमिी

Q.16)

ििदातमली और िमिे निम्ननलखित में से नकस र्ु ग से सोंबोंनधत स्थल थे?

बौद्ध धमय में ‘धमय चक्र प्रितयि’ निम्ननलखित में से

(a)

िि पाषाि काल (Neolithic Age)

(b)

मध्य पाषाि काल (Mesolithic Age)

(c)

ताम्र पाषाि काल (Chalcolithic Age)

(d)

उपर्ुयक्त में से कमई िहीों

नकसका प्रनतनिनधत्व करता है ?

Q.14)

(a)

बु द्ध की मृत्यु

(b)

बु द्ध कम ज्ञाि प्राखप्त

(c)

बु द्ध का प्रथम उपदे श

(d)

बु द्ध का महाि त्याग

Q.17)

उल्लेि बौद्ध ग्रन्थ ‘नमनलों द पन्हम’ में नमलता है ?

निम्ननलखित र्ु ग्मों में से कौि-सा/से सही सुमेनलत है /हैं? 1.

र्ि गु फाएों

:

किाय टक

2.

उदर्नगरी गु फाएों

:

नहमाचल प्रदे श

3.

बे डसा गु फाएों

महाराष्ट्र

िीचे नदए गए कूट का प्रर्मग कर सही उत्तर चु निए:

Q.15)

निम्ननलखित में से कौि-से र्ारतीर्-ग्रीक सम्राट का

Q.18)

(a)

नमिाों दर

(b)

रुद्रदमि

(c)

हे सानटर र्स

(d)

उपर्ुयक्त में से कमई िहीों

िै नदक काल की राििीनतक व्यिस्था की निम्ननलखित इकाइर्मों कम अिरमही क्रम में व्यिखस्थत कीनिए: 1.

निश

2.

िि

केिल 3

3.

ग्राम

1, 2 और 3

4.

राष्ट्र

5.

कुल

(a)

केिल 1 और 3

(b)

केिल 1 और 2

(c) (d)

निम्ननलखित में से नकस मौर्य सम्राट का उल्लेि, निनर्ि स्रमतमों में, मद्रासर, नसोंहसेि और अनमत्मचे टस (अनमत्घात) िै से िाममों के साथ र्ी नकर्ा गर्ा है ?

िीचे नदए गए कूट का प्रर्मग कर सही उत्तर चु निए: (a)

4-2-1-3-5

(a)

अशमक

(b)

2-3-1-5-4

(b)

चों द्रगु प्त

(c)

5-2-1-3–4

(c)

नबन्दु सार

(d)

3-2-4-1–5

(d)

दशरथ

RAUSIAS-TC19E1007

6

Q.12) Which

of

the

following

pairs

is

incorrectly matched?

belonging to which of the following

(a)

Nyaya- Gautama

(b)

Yoga- Patanjali

(c)

Sankhya- Badryana

(d)

Purva Mimansa - Jaimini

Q.13) Dharma

ages?

Chakra

Buddhism

Q.16) Navadatoli, and Jorwe were the sites

Pravartana

represents

which

of

in

(a)

Neolithic age

(b)

Mesolithic age

(c)

Chalcolithic age

(d)

None of the above

the

following? Q.17) Which of the following Indo-Greek kings

(a)

Death of Buddha

(b)

Enlightment of Buddha

is mentioned in the Buddhist text

(c)

First Sermon of Buddha

Milind-Panho?

(d)

Buddha’s Great Renunciation

(a)

Menander

(b)

Rudradaman

(c)

Hesatrius

(d)

None of the above

Q.14) Which of the following pairs is/are correctly matched? 1.

Bhaja Caves

:

Karnataka

2.

Udaygiri Caves

:

Himachal Pradesh

3.

Bedsa Caves

:

Maharashtra

Select the correct answer using the code given below:

Q.18) Arrange the following units of political organization of Vedic period in the descending order: 1.

Vis

2.

Jana

3 only

3.

Grama

1, 2 and 3

4.

Rashtra

5.

Kula

(a)

1 and 3 only

(b)

1 and 2 only

(c) (d)

Q.15) Which of the following Mauryan kings is also mentioned in different sources with the names like Madrasar, Simhasena

Select the correct answer using the code given below:

and Amitrochates?

(a)

4-2-1-3-5

(a)

Ashoka

(b)

2-3-1-5-4

(b)

Chandragupta

(c)

5-2-1-3-4

(c)

Bindusara

(d)

Dasaratha

(d)

3-2-4-1-5

RAUSIAS-TC19E1007

7

Q.19)

निम्ननलखित कथिमों पर निचार कीनिए: 1. 2.

Q.22)

महािीर के सोंदर्य में निम्ननलखित कथिमों में से कौि-से

हडप्पािानसर्मों िे ‘लािािदय ’ (Lapis Lazuli)

सही हैं ?

का लों बी दू री का व्यापार नकर्ा था।

1.

मेसमपमटानमर्ाई

ले ि

‘मेलुहा’

के

साथ

िे मािते थे, नक ईश्वर िे इस निश्व का निमाय ि नकर्ा है तथा इस पर निर्ोंत्ि रिता है ।

व्यापाररक सोंबोंधमों कम सोंदनर्यत करते हैं , िम नसोंधु

2.

क्षेत् कम नदर्ा गर्ा प्राचीि िाम था।

उन्हमोंिे सर्ी िस्तु ओ,ों सिीि अथिा नििीि, कम चे तिा के निनर्ि अोंशमों से सोंपि मािा था।

उपर्ुयक्त कथिमों में से कौि-सा/से सही है /हैं ?

3.

उन्हमोंिे िे दमों के प्रानधकार कम अस्वीकार कर

(a)

केिल 1

नदर्ा था, तथा िै नदक कमयकाोंड और ब्राह्मि

(b)

केिल 2

िचयस्व पर आपनत्त िताई थी।

(c)

1 और 2 दमिमों

(d)

ि तम 1, ि ही 2

4.

उन्हमोंिे उच्चतम आध्याखिक अिस्था की प्राखप्त के नलए गों र्ीर तपचर्ाय एिों अत्यनधक तपस्या का समथयि नकर्ा था।

Q.20)

मौर्य काल के सोंदर्य में निम्ननलखित कथिमों पर निचार

िीचे नदए गए कूट का प्रर्मग कर सही उत्तर चु निए:

कीनिए:

(a)

केिल 1 और 2

(b)

केिल 1, 2 और 3

(c)

केिल 2, 3 और 4

(d)

1, 2, 3 और 4

1.

कौनटल् के अथयशास्त्र के अिुसार, कृनष कार्ों में गु लाममों का उपर्मग मौर्य काल का एक अद् र्ुत सामानिक निकास था।

2.

मेगस्थिीज़ कहता है , नक उसिे र्ारत में नकसी गु लाम कम िहीों दे िा था। Q.23)

उपर्ुयक्त कथिमों में से कौि-सा/से सही है /हैं ?

Q.21)

(a)

केिल 1

(b)

केिल 2

(c)

1 और 2 दमिमों

(d)

ि तम 1, ि ही 2

गु प्त काल के सोंदर्य में निम्ननलखित कथिमों में से कौिसा/से सही है /हैं? 1.

शू द्रक िे शू न्य प्रिाली तथा दशमलि प्रिाली, दमिमों के बारे में िािकारी प्रदनशि॔त की है ।

2.

इलाहाबाद निले से प्राप्त गु प्त नशलाले िमों से पता चलता है , नक र्ारत में पााँचिीों शताब्दी की शु रुआत में दशमलि प्रिाली की िािकारी थी।

निम्ननलखित कथिमों पर निचार कीनिए: 1.

नलों गराि मोंनदर किाय टक में खस्थत है ।

2.

चालु क्य शासक

र्ीम

का

प्रनसद्ध

3. मोंत्ी

‘िास्तु पाल’एक ले िक, निद्वािमों का सोंरक्षक

एिों रममि निचारमों से प्रर्ानित था। 4.

उपर्ुयक्त कथिमों में से कौि-सा/से सही है /हैं ?

ह्वे ि त्ाों ग (Hsuan Tsang) कहता है , नक चाों डाल गााँ ि के बाहर रहते थे तथा िे माों स का

तथा माउों ट आबू में सुोंदर िै ि मोंनदर का निमाय ता था।

इस काल में सोंकनलत ‘रममक नसद्धान्त’ ग्रीक

कारमबार करते थे। िीचे नदए गए कूट का प्रर्मग कर सही उत्तर चु निए:

(a)

केिल 1

(a)

केिल 1

(b)

केिल 2

(b)

केिल 2, 3 और 4

(c)

1 और 2 दमिमों

(c)

केिल 2 और 3

(d)

ि तम 1, ि ही 2

(d)

केिल 1, 2 और 3

RAUSIAS-TC19E1007

8

Q.19) Consider the following statements: 1.

The Harappans carried on longdistance trade in lapis lazuli.

Q.22) Which of the following statements about Mahavira are correct? 1.

Mesopotamian records refer to trade relations with Meluha, which was the ancient name given to the Indus region.

He believed that God has created this world and exercises control over it.

2.

Which of the statements given above is/are correct?

He regarded all objects, animate or inanimate, as endowed with various degrees of consciousness.

3.

He rejected the authority of the Vedas and objected to the Vedic rituals and Brahmin supremacy.

4.

He advocated severe asceticism and extreme penance for the attainment of the highest spiritual state.

2.

(a)

1 only

(b)

2 only

(c)

Both 1 and 2

(d)

Neither 1 nor 2

Q.20) Consider the following statements regarding the Mauryan period: 1.

2.

According to the Arthashastra of Kautiliya, a striking social development of the Maurya period was the employment of slaves in agricultural operations. Megasthenes states that he did not notice any slaves in India.

Which of the statements given above is/are correct? (a)

1 only

(b)

2 only

(c)

Both 1 and 2

(d)

Neither 1 nor 2

Select the correct answer using the code given below: (a)

1 and 2 only

(b)

1, 2 and 3 only

(c)

2, 3 and 4 only

(d)

1, 2, 3 and 4

Q.23) Which of the following statements is/are correct regarding the Gupta period? 1.

Shudrka displays awareness of both the zero system and the decimal system.

2.

A Gupta inscription from Allahabad district suggests that the decimal system was known in India at the beginning of the fifth century.

3.

Romaka Siddhanta, compiled during this period, was influenced by Greek and Roman ideas.

4.

Hiuen Tsang states that the Chandalas lived outside the village and dealt in meat and flesh.

Q.21) Consider the following statements: 1.

Lingaraja temple is situated in Karnataka.

2.

Vastupala, the famous minister of the Chalukyan ruler Bhima, was a writer, a patron of scholars and the builder of the beautiful Jain temple at Mount Abu.

Which of the statements given above is/are correct? (a)

1 only

(b)

2 only

(c)

Both 1 and 2

(d)

Neither 1 nor 2

RAUSIAS-TC19E1007

Select the correct answer using the code given below: (a)

1 only

(b)

2, 3 and 4 only

(c)

2 and 3 only

(d)

1, 2 and 3 only

9

Q.24)

हषयिधय ि काल के सोंदर्य में निम्ननलखित कथिमों में से

Q.27)

कौि-सा/से सही है/हैं ? 1.

2.

सोंदर्य में निम्ननलखित कथिमों पर निचार कीनिए:

ह्वे ि त्ाों ग (Hsuan Tsang) से उसका सामाि

1.

समुदार् में तथा कार्य स्थल पर नहों सा से प्रर्ानित

िह (ह्वे ि त्ाों ग) शू द्रमों कम कृषक कहता था।

बच्चमों की सहार्ता करिे के नलए बिाए गए हैं । 2.

र्े केंद्र िानत, िगय, धमय, क्षेत्, र्ौि अनर्निन्यास

(a)

केिल 1

अथिा िै िानहक खस्थनत के अिपेक्ष, नहों सा से

(b)

केिल 2

प्रर्ानित 18 िषय से कम आर्ु के सर्ी व्यखक्तर्मों

(c)

1 और 2 दमिमों

की सहारा प्रदाि करें गे ।

(d)

ि तम 1, ि ही 2

उपर्ुयक्त कथिमों में से कौि-सा/से सही है /हैं ? (a)

केिल 1

हषयिधय ि िे निम्ननलखित में से कौि-सा/से िाटक नलिा

(b)

केिल 2

था/नलिे थे?

(c)

1 और 2 दमिमों

(d)

ि तम 1, ि ही 2

1.

रत्नािली

2.

नप्रर्दनशयका

3.

िागािोंद

िीचे नदए गए कूट का प्रर्मग कर सही उत्तर चु निए:

Q.28)

‘उद्यम सिी पमटय ल’ के सोंदर्य में निम्ननलखित कथिमों पर निचार कीनिए:

(a)

केिल 1

(b)

केिल 1 और 2

Ministry of Micro, Small and Medium

(c)

केिल 2 और 3

Enterprises - MSME) िे इस पमटय ल का

(d)

1, 2 और 3

प्रममचि नकर्ा है ।

1.

2. Q.26)

र्े नििी और सािय िनिक स्थािमों पर, पररिार में,

लू ट नलर्ा गर्ा था।

िीचे नदए गए कूट का प्रर्मग कर सही उत्तर चु निए:

Q.25)

‘िि स्टॉप सैन्टसय’ (One Stop Centres - OSCs) के

सूक्ष्म, लघु एिों मध्यम उद्यम मोंत्ालर् (The

र्ह पमटय ल मनहलाओों कम स्व-निश्वासी एिों आि-

कालाभ्र निद्रमह के सोंदर्य में निम्ननलखित कथिमों में से

निर्यर बिािे के नलए, कम लागत के उत्पादमों

कौि-सा/से सही है/हैं ?

और सेिाओों हे तु उद्यमशीलता का पमषि करिे

1.

2.

कालाभ्र निद्रमह र्ूनम अिुदानित ब्राह्मिमों के

तथा व्यिसार् के मॉडल के सृिि के नलए एक

निरुद्ध एक शखक्तशाली कृषक निद्रमह था।

िैटिकय है ।

कालाभ्र िै ि धमय के अिुर्ार्ी थे।

िीचे नदए गए कूट का प्रर्मग कर सही उत्तर चु निए:

उपर्ुयक्त कथिमों में से कौि-सा/से सही है /हैं ? (a)

केिल 1

(a)

केिल 1

(b)

केिल 2

(b)

केिल 2

(c)

1 और 2 दमिमों

(c)

1 और 2 दमिमों

(d)

ि तम 1, ि ही 2

(d)

ि तम 1, ि ही 2

RAUSIAS-TC19E1007

10

Q.24) Which

of

is/are

the

following

correct

statements

regarding

the

Q.27) Consider the following statements about

Harshavardhana period?

One Stop Centres (OSCs):

1.

1.

2.

Hsuan Tsang was robbed of his

These are intended to support

belongings.

children affected by violence, in

He (Hsuan Tsang) called Shudras

private and public spaces, within

agriculturists.

the family, community and at the workplace.

Select the correct answer using the code given below:

2.

These

centres

will

support

all

(a)

1 only

persons below 18 years of age

(b)

2 only

affected by violence, irrespective of

(c)

Both 1and 2

caste,

class,

(d)

Neither 1 nor 2

sexual

orientation

religion, or

region, marital

status. Q.25) Which of the following plays was/were

Which of the statements given above

authored by Harshavardhana?

is/are correct?

1.

Ratnavali

(a)

1 only

2.

Priyadarshika

(b)

2 only

3.

Nagananda

(c)

Both 1 and 2

(d)

Neither 1 nor 2

Select the correct answer using the code given below: (a)

1 only

(b)

1 and 2 only

the Udyam Sakhi Portal?

(c)

2 and 3 only

1.

(d)

1, 2 and 3

Q.28) Consider the following statements about The Ministry of Micro, Small and Medium

Enterprises

(MSME)

launched this portal. Q.26) Which is/are

of

the

correct

following about

statements

the

2.

2.

portal

nurturing

Kalabhra

revolt? 1.

The

is

a

network

entrepreneurship

for and

creating business models for low a

cost products and services in order

powerful peasant protest directed

to empower women and make

against the landed Brahmanas.

them

The Kalabhras were of Jainism

sufficient.

The

Kalabhra

revolt

was

persuasion.

self-reliant

self-

Which of the statements given above

Select the correct answer using the code

is/are correct?

given below:

(a)

1 only

(a)

1 only

(b)

2 only

(b)

2 only

(c)

Both 1 and 2

(c)

Both 1 and 2

(d)

Neither 1 nor 2

(d)

Neither 1 nor 2

RAUSIAS-TC19E1007

and

11

Q.29)

‘मनहला पुनलस स्वर्ों सेिी र्मििा’ (The Mahila Police

Volunteer

Scheme)

के

सोंदर्य

Q.31)

Festival of Innovation and Entrepreneurship

में

- FINE) के सोंदर्य में निम्ननलखित कथिमों पर निचार

निम्ननलखित कथिमों पर निचार कीनिए: 1.

‘फैखस्टिल ऑफ इिमिे शि एों ड एों टरप्रेन्यमरनशप’ (The

कीनिए:

मेघालर् इस र्मििा कम अपिािे िाला प्रथम

1.

राज्य था। 2.

सम्माि और पुरूस्कार प्रदाि करिे के नलए नित्त मोंत्ालर् की एक पहल है ।

र्ह र्मििा गाोंिमों में पुनलस प्रानधकाररर्मों और 2.

स्थािीर् समुदार्मों के बीच एक कडी के निमाय ि

2018 में ‘राष्ट्रीर् ििाचार फाउों डेशि’ (The National Innovation Foundation) तथा

की पररकल्पिा करती है ।

‘निज्ञाि

उपर्ुयक्त कथिमों में से कौि-सा/से सही है /हैं ?

Q.30)

र्ह ज़मीिी स्तर पर ििाचारमों कम मान्यता,

एिों

Department

केिल 1

(b)

केिल 2

(c)

1 और 2 दमिमों

(a)

केिल 1

(d)

ि तम 1, ि ही 2

(b)

केिल 2

(c)

1 और 2 दमिमों

(d)

ि तम 1, ि ही 2

1.

and

उपर्ुयक्त कथिमों में से कौि-सा/से सही है /हैं ?

Q.32)

‘इों पैखटों ग

ररसचय,

(IMPRINT-2)

इिमिेशि र्मििा’

एों ड

टै क्नमलॉिी-2

(The

Impacting

अिुसूनचत िििातीर् छात्मों के नलए एकलव्य

Research, Innovation and Technology – 2

मॉडल आिासीर् निद्यालर्मों कम सोंनिधाि के

Scheme) के सोंदर्य में निम्ननलखित कथिमों में से कौि-

अिुच्छेद 275(1) के अोंतगयत, अिुदाि के माध्यम से, राज्यमों/केंद्र शानसत प्रदे शमों में

सा/से सही है /हैं? 1.

अिुसूनचत िििानतर्मों की साक्षरता दर 1961

इसका नित्त पमषि मािि सों साधि निकास मोंत्ालर् तथा मनहला एिों बाल निकास मोंत्ालर्

स्थानपत नकर्ा गर्ा है । 2.

Science

(The

आर्मिि नकर्ा गर्ा था।

‘एकलव्य मॉडल आिासीर् निद्यालर् र्मििा’ (The

के सोंदर्य में निम्ननलखित कथिमों पर निचार कीनिए:

of

निर्ाग’

Technology) के सहर्मग से FINE का

(a)

Eklavya Model Residential School Scheme)

प्रौद्यमनगकी

के द्वारा नकर्ा िाएगा। 2.

इस कार्य क्रम का लक्ष्य ज्ञाि कम व्यिहार्य प्रौद्यमनगकी में पररिनतय त करके, राष्ट्र के समक्ष

से 2011 में कम हुई है ।

मौिू द प्रमुि अनर्र्ाों नत्की चु िौनतर्मों कम

उपर्ुयक्त कथिमों में से कौि-सा/से सही है /हैं ?

सम्बमनधत करिा है ।

(a)

केिल 1

िीचे नदए गए कूट का प्रर्मग कर सही उत्तर चु निए:

(b)

केिल 2

(a)

केिल 1

(c)

1 और 2 दमिमों

(b)

केिल 2

(d)

ि तम 1, ि ही 2

(c)

1 और 2 दमिमों

(d)

ि तम 1, ि ही 2

RAUSIAS-TC19E1007

12

Q.29) Consider the following statements about the Mahila Police Volunteer Scheme: 1.

2.

Meghalaya was the first state to

Q.31) Consider the following statements about the

Festival

of

Innovation

and

Entrepreneurship (FINE): 1.

It is an Initiative of the Ministry of

adopt this Scheme.

Finance to recognize, respect and

It envisages creation of a link

reward grassroots innovations.

between the police authorities and

2.

In 2018, the FINE was held in collaboration

with

Innovation

Foundation

and

Which of the statements given above

Department

of

and

is/are correct?

Technology.

the local communities in villages.

(a)

1 only

(b)

2 only

(c) (d)

Science

is/are correct? 1 only

Both 1 and 2

(b)

2 only

Neither 1 nor 2

(c)

Both 1 and 2

(d)

Neither 1 nor 2

the Eklavya Model Residential Schools

National

Which of the statements given above (a)

Q.30) Consider the following statements about

the

Q.32) Which is/are

of

the

correct

following about

statements

the

Impacting

Scheme:

Research, Innovation and Technology

1.

(IMPRINT)-2 Scheme?

Eklavya Model Residential Schools (EMRSs) for ST students are set up

1.

of Human Resource Development

in States / UTs with provisioning

(MHRD)

of funds through “Grants under Article 275(1) of the Constitution”. 2.

2.

(b)

2 only

(c) (d)

all

of

major

engineering

translating knowledge into viable technology. Select the correct answer using the code given below: (a)

1 only

(b)

2 only

Both 1 and 2

(c)

Both 1 and 2

Neither 1 nor 2

(d)

Neither 1 nor 2

RAUSIAS-TC19E1007

Ministry

challenges faced by the nation by

Which of the statements given above

1 only

the

The aim of the programme is to address

decreased from 1961 to 2011.

(a)

and

Women and Child Development.

The literacy rate of the STs has

is/are correct?

This will be funded by the Ministry

13

Q.33)

‘आर्ुष्माि

र्ारत

-

राष्ट्रीर्

स्वास्थ्य

सुरक्षा

Q.35)

‘राष्ट्रीर् पिि-सौर हाइनब्रड िीनत’ (The National

र्मििा’(Ayushman Bharat – National Health

Wind-Solar Hybrid Policy) के सोंदर्य में

Protection Scheme – ‘AB – NHPS’) के सोंदर्य में

निम्ननलखित कथिमों पर निचार कीनिए:

निम्ननलखित कथिमों में से कौि-सा/से सही है /हैं? 1.

1.

र्ह र्मििा निधय ि, िों नचत ग्रामीि पररिारमों तथा

र्ूनम के प्रर्ािी उपर्मग के नलए निशाल नग्रड से

पहचाि की गई व्यािसानर्क श्रेिी के शहरी

िु डे हुए पिि-सौर पी.िी. हाइनब्रड प्रिानलर्मों

श्रनमकमों के पररिारमों के नलए लनक्षत है । 2.

कम बढािा दे िे हे तु एक ढाों चा प्रदाि करिा है ।

प्रत्ये क सूचीबद्ध अस्पताल में एक 'आर्ुष्माि 2.

नमत्' हमगा, िम मरीज़मों की सहार्ता करे गा, तथा

इस िीनत का लक्ष्य ििीकरिीर् निद् र्ुत् उत्पादि में पररितय िशीलता कम कम करिा है ।

लार्ानथयर्मों और अस्पताल के मध्य समन्वर्

3.

इस िीनत का उद्दे श्य प्रसारि अिसोंरचिा एिों

स्थानपत करे गा।

उपर्ुयक्त कथिमों में से कौि-सा/से सही है /हैं ?

इस र्मििा के अोंतगय त लार् आिों नटत करिे के

(a)

केिल 1

(b)

केिल 2

(c)

1 और 2 दमिमों

(d)

ि तम 1, ि ही 2

नलए पररिार के आकार तथा आर्ु पर कमई सीमा िहीों हमगी। िीचे नदए गए कूट का प्रर्मग कर सही उत्तर चु निए: (a)

केिल 2

(b)

केिल 1 और 3

(c)

केिल 1 और 2

(d)

1, 2 और 3

Q.36)

‘हररत क्राों नत - कृषमिानत र्मििा’ (The Green Revolution – Krishonnati Yojana) के सोंदर्य में निम्ननलखित कथिमों पर निचार कीनिए:

Q.34)

‘है म्मर’ (HAMMER – The Hypervelocity

1.

र्ह एक अम्ब्रै ला र्मििा है , निसका लक्ष्य कृनष

Asteroid Mitigation Mission for Emergency

तथा सोंबद्ध क्षेत् का समग्र और िै ज्ञानिक तरीके

Response vehicle) के सोंदर्य में निम्ननलखित कथिमों

से निकास करिा है ।

पर निचार कीनिए: 1.

र्ह हमारे ग्रह की सुरक्षा के नलए ितरा उत्पि

2.

Food Security Mission – NFSM) तथा

करिे िाली अोंतररक्ष चट्टािमों कम िष्ट् करिे का

‘बागिािी के एकीकृत निकास के नलए

‘इसरम’ (ISRO) का प्रमुि अनर्र्ाि है । 2.

अनर्र्ाि’ (The Mission for Integrated

है म्मर कम अोंतररक्ष चट्टािमों कम तमडिे के नलए

Development of Horticulture – MIDH)

गनति प्रर्ािक अथिा नकसी अन्य परमािु

इस र्मििा के घटक हैं ।

उपकरि के िाहक के रूप में प्रर्ु क्त नकर्ा िा सकता है । उपर्ुयक्त कथिमों में से कौि-सा/से सही है /हैं ?

‘राष्ट्रीर् िाद्य सुरक्षा अनर्र्ाि’ (The National

उपर्ुयक्त कथिमों में से कौि-सा/से सही है /हैं ? (a)

केिल 1

(a)

केिल 1

(b)

केिल 2

(b)

केिल 2

(c)

1 और 2 दमिमों

(c)

1 और 2 दमिमों ि तम 1, ि ही 2

(d)

ि तम 1, ि ही 2

(d)

RAUSIAS-TC19E1007

14

Q.33) Which

of

the

following

statements

is/are correct about the Ayushman Bharat-National

Health

Protection

Scheme (AB-NHPS): 1.

the National Wind-solar Hybrid Policy: 1.

rural

families

of large grid connected wind-solar

and

identified occupational category of

PV (Photovoltaic) hybrid system for

urban workers' families.

efficient utilization of transmission

Each empanelled hospital will have

infrastructure and land.

an 'Ayushman Mitra' to assist patients and will coordinate with

2.

There will be no cap on the family

The policy aims to reduce the variability

beneficiaries and the hospital. 3.

The objective of the policy is to provide a framework for promotion

The Scheme is targeted at poor, deprived

2.

Q.35) Consider the following statements about

in

renewable

power

generation.

size and age in order to allocate

Which of the statements given above

benefits under the AB-NHPS.

is/are correct?

Select the correct answer using the code

(a)

1 only

given below:

(b)

2 only

(c)

Both 1 and 2

(d)

Neither 1 nor 2

(a)

2 only

(b)

1 and 3 only

(c)

1 and 2 only

(d)

1, 2 and 3 Q.36) Consider the following statements about

Q.34) Consider the following statements about HAMMER Mitigation

(Hypervelocity Mission

for

Asteroid Emergency

Response vehicle): 1.

the

1.

—Krishonnati

It is an umbrella Scheme that aims to develop agriculture and allied sectors in a holistic and

to shunt or blow up threatening

scientific manner.

space rocks in order to protect our 2.

Revolution

Yojana’:

It is ISRO’s key mission that plans

planet.

‘Green

2.

The

National

Food

Security

HAMMER can be used as a kinetic

Mission (NFSM) and the Mission

impactor or as a carrier for some

for

other nuclear device which can do

Horticulture

the same job to blow up space rocks. Which of the statements given above is/are correct?

Integrated

(MIDH)

are

of the

components of the Scheme. Which of the statements given above is/are correct?

(a)

1 only

(a)

1 only

(b)

2 only

(b)

2 only

(c)

Both 1 and 2

(c)

Both 1 and 2

(d)

Neither 1 nor 2

(d)

Neither 1 nor 2

RAUSIAS-TC19E1007

Development

15

Q.37)

'हररत कौशल निकास कार्यक्रम’ (The Green Skill Development

Programme)

के

सोंदर्य

Q.39)

Mission) निम्ननलखित में से नकस मोंत्ालर् के द्वारा

में

आरम्भ नकर्ा गर्ा है ?

निम्ननलखित कथिमों पर निचार कीनिए: 1.

इस कार्य क्रम का लक्ष्य तकिीकी िािकारी एिों धारिीर् निकास के प्रनत प्रनतबद्धता रििे िाले हररत कुशल श्रनमकमों का निकास करिा है ।

2.

र्ह कार्यक्रम ‘राष्ट्रीर् कौशल निकास एिें सी’ (The

National

Agency)

के

Skill

परामशय

Development से

‘सौर चरिा अनर्र्ाि’ (The Solar Charkha

‘ििीि

एिों

Q.40)

(a)

सूक्ष्म, लघु एिों मध्यम उद्यम मोंत्ालर्

(b)

रक्षा मोंत्ालर्

(c)

गृ ह मोंत्ालर्

(d)

नित्त मोंत्ालर्

‘प्रधाि मोंत्ी मातृ िों दिा र्मििा’ (Pradhan Mantri Matru Vandana Yojana – PMMVY) के सोंदर्य

ििीकरिीर् ऊिाय मोंत्ालर्’ (The Ministry

में निम्ननलखित कथिमों पर निचार कीनिए:

of New and Renewable Energy) के द्वारा

1.

उपक्रममों में, सर्ी गर्यिती मनहलाओों और

निकनसत नकर्ा गर्ा है ।

स्तिपाि करािे िाली माताओों कम पररिार के

उपर्ुयक्त कथिमों में से कौि-सा/से सही है /हैं ?

Q.38)

(a)

केिल 1

(b)

केिल 2

(c)

1 और 2 दमिमों

(d)

ि तम 1, ि ही 2

सर्ी िीनित बच्चमों के नलए मातृ त्व लार् उपलब्ध हैं । 2.

उपर्ुयक्त कथिमों में से कौि-सा/से सही है /हैं ?

Vaya Vandana Yojana – PMVVY) के सोंदर्य में निम्ननलखित कथिमों पर निचार कीनिए: र्ह र्मििा, र्ारतीर् िीिि बीमा निगम कम

Q.41)

(a)

केिल 1

(b)

केिल 2

(c)

1 और 2 दमिमों

(d)

ि तम 1, ि ही 2

‘अटल र्ूिल र्मििा’ (Atal Bhujal Yojana –

सरकार की गारों टी के आधार पर सदस्यता

ABHY) के सोंदर्य में निम्ननलखित कथिमों पर निचार

रानश से िु डी हुई सुनिनित पेंशि/ररटिय (लार्)

कीनिए:

के प्रािधाि के माध्यम से, िररष्ठ िागररकमों के

1.

र्ू-िल तिािग्रस्त क्षेत्मों में समुदानर्क र्ागीदारी

र्ह र्मििा 10 िषों के नलए 8% प्रनत िषय का सुनिनित ररटिय (लार्) प्रदाि करती है ।

उपर्ुयक्त कथिमों में से कौि-सा/से सही है /हैं ?

िल सोंसाधि मोंत्ालर् के द्वारा आरम्भ की गई इस र्मििा का लक्ष्य चर्नित अनत-शमनषत एिों

नलए िृ द्धािस्था आर् सुरक्षा सुनिनित करती है । 2.

इस र्मििा के अोंतगयत पात् लार्ानथयर्मों कम 5,000 रु प्राप्त हमोंगें।

‘प्रधाि मोंत्ी िर् िों दिा र्मििा’ (Pradhan Mantri

1.

इस र्मििा के अोंतगय त, साियिनिक क्षेत् के

से धारिीर् र्ू-िल प्रबोंधि करिा है । 2.

इसे निश्व बैं क की सहार्ता से लागू नकर्ा िािा प्रस्तानित है ।

(a)

केिल 1

उपर्ुयक्त कथिमों में से कौि-सा/से सही है /हैं ?

(b)

केिल 2

(a)

केिल 1

(c)

1 और 2 दमिमों

(b)

केिल 2

(d)

ि तम 1, ि ही 2

(c)

1 और 2 दमिमों

(d)

ि तम 1, ि ही 2

RAUSIAS-TC19E1007

16

Q.37) Consider the following statements about the

Green

Skill

Development

Programme: 1.

The programme aims to develop green

(a)

The Ministry of Micro, Small and Medium Enterprises (MSME)

workers

having

(b)

The Ministry of Defence

knowledge

and

(c)

The Ministry of Home Affairs

sustainable

(d)

The Finance Ministry

skilled

technical

Q.39) The Solar Charkha Mission is launched by which of the following Ministries?

commitment

to

development. 2.

It has been developed by the Ministry of New and Renewable Energy in consultation with the National Skill Development Agency

Q.40) Consider the following statements about the Pradhan Mantri Matru Vandana Yojana (PMMVY): 1.

Under this Scheme, maternity benefits are available to all pregnant women and lactating mothers in Public Sector Undertaking for any number of living children of the family.

2.

Eligible beneficiaries will get Rs. 5,000 under PMMVY.

(NSDA). Which of the statements given above is/are correct? (a)

1 only

(b)

2 only

(c)

Both 1 and 2

(d)

Neither 1 nor 2

Which of the statements given above is/are correct?

Q.38) Consider the following statements about the

Pradhan Mantri Vaya Vandana

Yojana (PMVVY): 1.

The

Scheme

enables

old

provision

pension/return subscription

of

linked

amount

1 only

(b)

2 only

(c)

Both 1 and 2

(d)

Neither 1 nor 2

age

income security for senior citizens through

(a)

assured to

the

based

Q.41) Consider the following statements about the Atal Bhujal Yojana (ABHY): 1.

Launched by Ministry of Water Resources the Scheme is aimed at sustainable ground water management with community participation in select overexploited and ground water stressed areas.

2.

It is proposed to be implemented with the World Bank’s assistance.

on

government guarantee to the Life Insurance

Corporation

of

India

(LIC). 2.

The Scheme provides an assured return of 8% per annum for 10 years.

Which of the statements given above is/are correct?

Which of the statements given above is/are correct?

(a)

1 only

(a)

1 only

(b)

2 only

(b)

2 only

(c)

Both 1 and 2

(c)

Both 1 and 2

(d)

Neither 1 nor 2

(d)

Neither 1 nor 2

RAUSIAS-TC19E1007

17

Q.42)

‘स्विल र्मििा’ (The Swajal Scheme) के सोंदर्य में

Q.44)

Nitrogen Initiative – INI) के सोंदर्य में

निम्ननलखित कथिमों में से कौि-सा/से सही है /हैं? 1.

निम्ननलखित कथिमों में से कौि-सा/से सही है /हैं?

र्ह र्मििा ‘पेर् िल एिों स्वच्छता मोंत्ालर्’ के

1.

द्वारा कार्ाय खन्वत की गई है । 2.

‘अोंतराय ष्ट्रीर् िाइटर मिि पहल’ (The International

एििार्रमैंट’ (The Scientific Committee

‘स्विल र्मििा’ कम ग्रामीि क्षेत्मों में लमगमों कम

on Problems of the Environment -

पेर्िल की सतत पहुाँ च प्रदाि करिे के नलए,

3.

SCOPE) तथा ‘इों टरिैशिल िीर्मस्फेर्र-

एक माों ग आधाररत एिों समुदार् केंनद्रत

बार्मस्फेर्र प्रमग्राम’ (The International

कार्यक्रम के रूप में तै र्ार नकर्ा गर्ा है ।

Geosphere-Biosphere Programme) के

इस र्मििा का लक्ष्य गाोंिमों कम सौर ऊिाय के

प्रर्मिि के अोंतगयत स्थानपत है । 2.

द्वारा सोंचानलत पाइप िलापूनतय उपलब्ध करािा

अोंतराय ष्ट्रीर् िाइटर मिि पहल हर तीि साल में एक सम्मेलि आर्मनित करता है ।

है । 3.

िीचे नदए गए कूट का प्रर्मग कर सही उत्तर चु निए:

Q.43)

र्ह ‘साइों नटनफक कमेटी ऑि प्रॉब्लम्स ऑफ़ द

इस पहल का लक्ष्य धारिीर् िाद्य उत्पादि में िाइटर मिि की लार्कारी र्ूनमका का बे हतर

(a)

केिल 2

(b)

केिल 1 और 3

िीचे नदए गए कूट का प्रर्मग कर सही उत्तर चु निए:

(c)

केिल 1 और 2

(a)

केिल 3

(d)

1, 2 और 3

(b)

केिल 1 और 3

(c)

केिल 1 और 2

(d)

1, 2 और 3

अिुकूलि करिा है ।

‘पररिेश’ (PARIVESH) के सोंदर्य में निम्ननलखित कथिमों में से कौि-सा/से सही है /हैं ? 1.

र्ह एक एकीकृत पर्ाय िरि प्रबोंधि प्रिाली है ।

2.

इस प्रिाली कम ‘राष्ट्रीर् सूचिा निज्ञाि केंद्र’ (The National Informatics Centre NIC) की तकिीकी सहार्ता से, ‘ििीि एिों ििीकरिीर् ऊिाय मोंत्ालर्’ के द्वारा निकनसत

3.

Q.45)

‘एि.ए.बी.एच. निमाय ि पहल’ (NABH Nirman Initiative) के सोंदर्य में निम्ननलखित कथिमों में से कौि-सा/से सही है/हैं ? 1.

इस र्मििा का लक्ष्य हिाई अड्मों की क्षमता का निस्तार करिा है ।

2.

र्ह केिल सरकार के स्वानमत्व िाले ‘र्ारतीर्

तथा पमनषत नकर्ा गर्ा है ।

निमािपत्ति

र्ह प्रिाली अनधकाररर्मों की सहार्ता करती है ,

Authority of India – AAI) के द्वारा हिाई

क्यमोंनक इसमें केंद्र, राज्य और निला स्तर,

अड्मों के उिर्ि में नकए िािे िाले नििे शमों का

प्रत्ये क की स्वीकृनत के नलए ‘एकल-खिडकी

गठि करती है ।

प्रिाली’ (Single-Window System) है । िीचे नदए गए कूट का प्रर्मग कर सही उत्तर चु निए: (a)

केिल 1

(b)

केिल 1 और 3

(c) (d)

3.

प्रानधकरि’

Airport

र्ह पहल छमटे कस्मों और शहरमों कम िमडिे तथा पर्य टि में िृ खद्ध करिे में सहार्ता करे गी।

िीचे नदए गए कूट का प्रर्मग कर सही उत्तर चु निए: (a)

केिल 2

(b)

केिल 1 और 3

केिल 1 और 2

(c)

केिल 1 और 2

1, 2 और 3

(d)

1, 2 और 3

RAUSIAS-TC19E1007

(The

18

Q.42) Which

of

is/are

the

correct

following about

statements the

Swajal

Q.44) Which is/are

of

the

correct

following about

Scheme?

Nitrogen Initiative(INI)?

1.

1.

2.

This Scheme is implemented by

International

It is an international programme,

the Ministry of Drinking Water and

set up under sponsorship of the

Sanitation.

Scientific Committee on Problems

“Swajal” that is designed as a

of the Environment (SCOPE) and

demand driven and community

from the International Geosphere-

centered programme to provide

Biosphere Programme (IGBP).

sustainable

access

to

drinking

2.

water to people in rural areas. 3.

statements

The INI holds a conference every three years.

The Scheme will aim to provide

3.

The aim of the Initiative is to

villages with piped water supply

optimize nitrogen’s beneficial role

powered by solar energy.

in sustainable food production.

Select the correct answer using the code

Select the correct answer using the code

given below:

given below:

(a)

2 only

(a)

3 only

(b)

1 and 3 only

(b)

1 and 3 only

(c)

1 and 2 only

(c)

1 and 2 only

(d)

1, 2 and 3

(d)

1, 2 and 3

Q.43) Which

of

the

following

statements

Q.45) Which

of

is/are correct about PARIVESH?

is/are

1.

It is an Integrated Environmental

Initiative?

Management System.

1.

2.

and

3.

Renewable

Energy,

following

statements

correct about NABH Nirman

This Scheme is aimed towards expanding airport capacity.

The system has been developed and hosted by the Ministry of New

the

2.

It constitutes investments to be

with

made in airport upgrade by only

technical support from National

the state-owned Airports Authority

Informatics Centre, (NIC).

of India (AAI).

The system helps authorities as it

3.

The Initiative will help to connect

has a single window system each

smaller

for the central, state and district

increase tourism.

level clearances.

towns

cities

and

Select the correct answer using the code

Select the correct answer using the code

given below:

given below:

(a)

2 only

(a)

1 only

(b)

1 and 3 only

(b)

1 and 3 only

(c)

1 and 2 only

(c)

1 and 2 only

(d)

1, 2 and 3

(d)

1, 2 and 3

RAUSIAS-TC19E1007

and

19

Q.46)

‘प्रधाि मोंत्ी अिदाता आर् सोंरक्षि अनर्र्ाि’ (Pradhan Manrti Annadata Aay SanraksHan

Q.48)

कथिमों पर निचार कीनिए:

Abhiyan – PM-AASHA) के सोंदर्य में निम्ननलखित

1.

कथिमों में से कौि-सा/से सही है /हैं ? 1.

र्ाषाओों की सामग्री शीघ्रता से ऑिलाइि

प्रौद्यमनगनकर्मों की उपलब्धता सुनिनित करिा

प्रकानशत करिे में सक्षम करे गा। 2.

‘मूल् समथयि र्मििा’ (The Price Support

द्वारा, मािि सोंसाधि निकास मोंत्ालर् की

र्मििा’ (The Price Deficiency Payment

साझेदारी में आरम्भ की गई थी। उपर्ुयक्त कथिमों में से कौि-सा/से सही है /हैं ?

घटक हैं । ‘मूल् समथयि र्मििा’ में राज्य सरकारमों की सनक्रर् र्ूनमका के साथ केंद्रीर् िमडल एिें नसर्मों के द्वारा दालमों, नतलहिमों और िमपरा की र्ौनतक प्राखप्त की िाएगी। िीचे नदए गए कूट का प्रर्मग कर सही उत्तर चु निए:

Q.47)

(a)

केिल 1

(b)

केिल 2 और 3

(c)

केिल 1 और 3

(d)

1, 2 और 3

‘बाों ध पुििाय स एिों सुधार पररर्मििा’ (Dam Rehabilitation and Improvement Project – DRIP) के सोंदर्य में निम्ननलखित कथिमों में से कौि-

Q.49)

(a)

केिल 1

(b)

केिल 2

(c)

1 और 2 दमिमों

(d)

ि तम 1, ि ही 2

‘सौर्ाग्य – प्रधाि मोंत्ी सहि नबिली हर घर र्मििा' (Saubhagya - Pradhan Mantri Sahaj Bijli Har Ghar Yojana) के सोंदर्य में निम्ननलखित कथिमों में से कौि-सा/से सही है/हैं ? 1.

इस पररर्मििा कम 2012 में िल सोंसाधि, िदी

2.

निकास एिों गों गा सोंरक्षि मोंत्ालर् के द्वारा

कर नलर्ा है ।

सुधार के उद्दे श्य से आरम्भ नकर्ा गर्ा था। इस पररर्मििा कम ‘एनशर्ाई निकास बैं क’ (The Asian Development Bank – ADB)

3.

Connectivity) तथा नबिली किैक्शि प्रदाि

र्ह पररर्मििा र्ारत में मौिूदा बाोंधमों के केिल

करिे की पररकल्पिा करती है ।

पुििाय स से सोंबोंनधत है । िीचे नदए गए कूट का प्रर्मग कर सही उत्तर चु निए: (a)

केिल 1

(b)

केिल 1 और 3

(c) (d)

र्ह र्मििा केिल ग्रामीि क्षेत्मों के सर्ी शे ष घरमों में ‘अोंनतम मील सोंपकय’ (The Last Mile

की सहार्ता से आरम्भ नकर्ा गर्ा था। 3.

मध्य प्रदे श, नत्पुरा तथा नबहार िे इस र्मििा के अोंतगयत 100% निद् र्ु तीकरि का लक्ष्य प्राप्त

चर्नित बाों धमों की सुरक्षा और पररचालि में 2.

इसे 2017 में दे श के सर्ी शे ष घरमों में नबिली पहुों चािे के उद्दे श्य से आरम्भ नकर्ा गर्ा था।

सा/से सही है /हैं? 1.

र्ह पहल ‘माइक्रमसॉफ्ट’ (Microsoft) के

Scheme – PSS) तथा ‘मूल् कमी र्ुगताि Scheme – PDPS) इस र्मििा के प्रमुि 3.

र्ह एक ऐसा मोंच है , िम कोंपनिर्मों कम क्षेत्ीर्

इस र्मििा का लक्ष्य नकसािमों कम कुशल आगत है ।

2.

‘ििले िा’ (Navlekha) के सन्दर्य में निम्ननलखित

िीचे नदए गए कूट का प्रर्मग कर सही उत्तर चु निए: (a)

केिल 2

केिल 1 और 2

(b)

केिल 1 और 3

1, 2 और 3

(c)

केिल 1 और 2

(d)

1, 2 और 3

RAUSIAS-TC19E1007

20

Q.46) Which of the following statements is/are correct about “Pradhan Mantri Annadata Aay SanraksHan Abhiyan” (PM-AASHA)? 1.

2.

3.

The Scheme is aimed at ensuring the availability of efficient input technologies to the farmers. Price Support Scheme (PSS) and Price Deficiency Payment Scheme (PDPS) form key components of the Scheme. In Price Support Scheme (PSS), physical procurement of pulses, oilseeds and the Copra will be done by Central Nodal Agencies with proactive role of the state governments.

Select the correct answer using the code given below: (a)

1 only

(b)

2 and 3 only

(c)

1 and 3 only

(d)

1, 2 and 3

Q.48) Consider the following statements about Navlekha: 1.

regional languages content firms to publish online quickly. 2.

2.

3.

This Project was launched in 2012 by Ministry of Water Resources, River Development and Ganga Rejuvenation with an objective to improve safety and operational performance of selected dams.

Ministry

Select the correct answer using the code given below: (a)

1 only

(b)

1 and 3 only

(c)

1 and 2 only

(d)

1, 2 and 3

RAUSIAS-TC19E1007

of

Human

Resource

Development. Which of the statements given above is/are correct? (a)

1 only

(b)

2 only

(c)

Both 1 and 2

(d)

Neither 1 nor 2

Q.49) Which is/are

of

the

following

correct

about

statements

Saubhagya



‘Pradhan Mantri Sahaj Bijli Har Ghar Yojana’? 1.

It was launched in 2017 with the objective

to

provide

access

to

electricity to all the remaining households in the country. 2.

Madhya

Pradesh,

Tripura

and

Bihar have achieved the goal of 100%

electrification

under

the

Scheme. 3.

The Scheme envisages to provide last

The Project was launched with the Assistance of ADB (Asian Development Bank). The Project deals with only rehabilitation of existing dams in India.

The Initiative was launched by Microsoft in partnership with the

Q.47) Which of the following statements is/are correct about Dam Rehabilitation and Improvement Project (DRIP)? 1.

It is a platform that will enable

mile

connectivity

electricity

connections

remaining

households

and to

in

all rural

areas only. Select the correct answer using the code given below: (a)

2 only

(b)

1 and 3 only

(c)

1 and 2 only

(d)

1, 2 and 3

21

Q.50)

‘नडिी र्ात्ा पहल’ (The Digi Yatra Initiative) के

Q.52)

महापाषाि सोंस्कृनत के सोंदर्य में निम्ननलखित कथिमों

सोंदर्य में निम्ननलखित कथिमों में से कौि-सा/से सही

पर निचार कीनिए:

है /हैं?

1.

1.

नज़क्र नमलता है ।

र्ह िागररक उड्र्ि मोंत्ालर् की एक पहल है , 2.

िम कागज़-निहीि और परे शािी मुक्त हिाई

3.

काले और लाल मृदर्ाों ड उिके मध्य लमकनप्रर् प्रतीत हमते हैं ।

र्ात्ा कम बढािा दे ती है । 2.

तनमल सोंगम सानहत्य में महापाषाि सोंस्कृनत का

उपर्ुयक्त कथिमों में से कौि-सा/से सही है /हैं ?

इस पहल के अोंतगयत र्ानत्र्मों के नलए ‘एक केंद्रीकृत पोंिीकरि प्रिाली’ (A Centralized

(a)

केिल 1

Registration System) हमगी तथा प्रत्ये क

(b)

केिल 2

र्ात्ी कम नटकट बु क करिे पर ‘एक र्ू निक

(c)

1 और 2 दमिमों

आई.डी.’ (A Unique ID) नमले गी।

(d)

ि तम 1, ि ही 2

इस पहल के अोंतगयत बु नकोंग के समर् ‘आधार’ (Aadhaar)

कम

एर्रलाइों स

से

िमडिा

Q.53)

में से नकस राज्य में खस्थत है?

अनििार्य है । िीचे नदए गए कूट का प्रर्मग कर सही उत्तर चु निए: (a)

केिल 2

(b)

केिल 1 और 3

(c)

केिल 1 और 2

(d)

1, 2 और 3

‘हे नमस मठ’ (The Hemis Monastery) निम्ननलखित

Q.54)

(a)

िम्मू-कश्मीर

(b)

नहमाचल प्रदे श

(c)

पनिम बों गाल

(d)

तनमलिाडु

‘मुनज़ररस निरासत पररर्मििा' (The Muziris Heritage Project) के सोंदर्य में निम्ननलखित कथिमों

Q.51)

मौर्य साम्राज्य के सोंदर्य में निम्ननलखित कथिमों पर

में से कौि-सा/से सही है /हैं ?

निचार कीनिए:

1.

1.

अशमक िे मनहलाओों सनहत, निनर्ि सामानिक

के ऐनतहानसक एिों साों स्कृनतक महत्व कम

समूहमों के मध्य धमय के प्रचार के नलए

पुिःस्थानपत करिे के नलए मुनज़ररस निरासत पररर्मििा की शु रुआत की है ।

धम्ममहामन्त्मों की निर्ु खक्त की थी। 2.

केरल सरकार िे मुनज़ररस के प्रनसद्ध बों दरगाह

चों द्रगु प्त मौर्य िे अपिे साम्राज्य में न्यार् के प्रशासि के नलए रािु कमों कम निर्ु क्त नकर्ा था।

उपर्ुयक्त कथिमों में से कौि-सा/से सही है /हैं ? (a)

केिल 1

(b)

केिल 2

(c)

1 और 2 दमिमों

(d)

ि तम 1, ि ही 2

RAUSIAS-TC19E1007

2.

र्ह केरल सरकार की प्रथम हररत पररर्मििा है ।

िीचे नदए गए कूट का प्रर्मग कर सही उत्तर चु निए: (a)

केिल 1

(b)

केिल 2

(c)

1 और 2 दमिमों

(d)

ि तम 1, ि ही 2

22

Q.50) Which is/are

of

the

following

correct

about

statements Digi

Yatra

Initiative? 1.

Q.52) Consider the following statements about the Megalithic culture: 1.

This is an Initiative of by the

culture

Ministry

literature.

of

Civil

Aviation

that

promotes paperless and hassle-

2.

free air travel. 2.

in

Tamil

Sangam

Black and red ware seems to have been popular with them.

Under the Initiative a centralized

Which of the statements given above

registration system for passengers

is/are correct?

would be there and each of them would get a unique ID on booking the tickets. 3.

There are references to megalithic

Linking of Aadhaar to airlines at the time of booking is mandatory

(a)

1 only

(b)

2 only

(c)

Both 1 and 2

(d)

Neither 1 nor 2

under the Initiative. Select the correct answer using the code

Q.53) The Hemis Monastery is situated in

given below:

which of the following states?

(a)

2 only

(a)

Jammu and Kashmir

(b)

1 and 3 only

(b)

Himachal Pradesh

(c)

1 and 2 only

(c)

West Bengal

(d)

1, 2 and 3

(d)

Tamil Nadu

Q.51) Consider the following statements about

Q.54) Which

of

the

following

the Mauryan Empire:

is/are

1.

Ashoka appointed dhammamah-

Heritage Project?

amantras to propagate dharma

1.

among 2.

various

social

groups

correct

about

statements

the

Muziris

The Government of Kerala has initiated

the

Muziris

Heritage

including women.

Project to reinstate the historical

Chandragupta Maurya appointed

and cultural significance of the

Rajukas for the administration of

legendary port of Muziris.

justice in his empire.

2.

Which of the statements given above

It is the first Green Project of the Government of Kerala.

is/are correct?

Select the correct answer using the code

(a)

1 only

given below:

(b)

2 only

(a)

1 only

(c)

Both 1 and 2

(b)

2 only

(d)

Neither 1 nor 2

(c)

Both 1 and 2

(d)

Neither 1 nor 2

RAUSIAS-TC19E1007

23

Q.55) नलों गमद्भिमू नतय के सोंदर्य में निम्ननलखित कथिमों में से कौि-सा/से सही है /हैं ? 1.

2.

1.

नलस्ट’ (The IUCN Red List) में 'निकट

िाली 12िीों शताब्दी की एक ग्रेिाइट मू नतय है ।

ितरे में ’ (Near Threatened) के रूप में

र्ह चालु क्य िोंश से सोंबोंनधत है ।

सूचीबद्ध है ।

(a)

केिल 1

(b)

केिल 2

(c)

1 और 2 दमिमों

(d)

ि तम 1, ि ही 2

Q.56) निम्ननलखित कथिमों पर निचार कीनिए: र्ारत में कार् र्रत निदे शी बैंकमों की शािाओों सनहत, सर्ी िानिखज्यक बैंकमों, स्थािीर् क्षे त् बैंकमों तथा क्षे त्ीर् ग्रामीि बैंकमों कम ‘िमा बीमा एिों ऋि गारों टी निगम’ (The Deposit

2.

Corporation) के द्वारा समानिष्ट् नकर्ा

निगम के द्वारा समानिष्ट् िहीों नकए िाते हैं ।

केिल 1

(b)

केिल 2

(c) (d)

Wildlife

उपर्ुयक्त कथिमों में से कौि-सा/से सही है /हैं ? (a)

केिल 1

(b)

केिल 2

(c)

1 और 2 दमिमों

(d)

ि तम 1, ि ही 2

Q.58) 'नचनत्त/रों गे हुए सारस’ (Painted Stork) के सोंदर्य में निम्ननलखित कथिमों पर निचार कीनिए: 1.

र्ह ‘आई.र्ू.सी.एि. रे ड नलस्ट’ (The IUCN Red List) में 'निकट ितरे में ’ (Near Threatened) के रूप में सूचीबद्ध है ।

2.

र्ह ‘िन्यिीि सोंरक्षि अनधनिर्म, 1972’ (The Wildlife Protection Act, 1972) की अिु सूची-I के अोंतगयत सोंरनक्षत है ।

उपर्ुयक्त कथिमों में से कौि-सा/से सही है /हैं ? (a)

केिल 1

1 और 2 दमिमों

(b)

केिल 2

ि तम 1, ि ही 2

(c)

1 और 2 दमिमों

(d)

ि तम 1, ि ही 2

RAUSIAS-TC19E1007

(The

अोंतगयत सोंरनक्षत है ।

उपर्ुयक्त कथिमों में से कौि-सा/से सही है /हैं ? (a)

1972’

Protection Act, 1972) की अिु सूची-IV के

िाता है । सहकारी बैंक, िमा बीमा एिों ऋि गारों टी

नहमालर्ि नग्रफ़ॉि नगद्ध ‘िन्यिीि सोंरक्षि अनधनिर्म,

Insurance and Credit Guarantee

2.

नहमालर्ि नग्रफॉि नगद्ध ‘आई.र्ू.सी.एि. रे ड

र्ह र्गिाि नशि का प्रनतनष्ठत नचत्ि करिे

िीचे नदए गए कूट का प्रर्मग कर सही उत्तर चुनिए:

1.

Q.57) निम्ननलखित कथिमों पर निचार कीनिए:

24

Q.55) Which is/are

of

the

following

statements

about

Lingodh-

correct

Q.57) Consider the following statements: 1.

bhavamurti? 1.

It

is

as ‘Near Threatened’ in the IUCN a

12th

sculpture,

century

depicting

granite

an

It

is

related

to

the

Red list.

iconic 2.

representation of Lord Shiva. 2.

Himalayan Griffon Vulture is listed

protected in Schedule IV of the

Chalukya

dynasty.

Wildlife Protection Act 1972.

Select the correct answer using the code given below: (a)

1 only

(b)

2 only

(c)

Both 1 and 2

(d)

Neither 1 nor 2

It (Himalayan Griffon Vulture) is

Which of the statements given above is/are correct? (a)

1 only

(b)

2 only

(c)

Both 1 and 2

(d)

Neither 1 nor 2

Q.56) Consider the following statements: 1.

All commercial banks including the branches of foreign banks functioning in India, Local Area Banks and Regional Rural Banks are

covered

by

the

Q.58) With

1.

Co-operative

are

It is listed as ‘Near threatened’ in

2.

It is protected in Schedule I of the

not

and Credit Guarantee Corporation.

Which of the statements given above is/are correct?

Which of the statements given above

(a)

1 only

is/are correct?

(b)

2 only

(c)

Both 1 and 2

(d)

Neither 1 nor 2

1 only

(b)

2 only

(c)

Both 1 and 2

(d)

Neither 1 nor 2

RAUSIAS-TC19E1007

storks’,

Wildlife Protection Act 1972. Banks

covered by the Deposit Insurance

(a)

‘Painted

IUCN red list.

Corporation. 2.

to

consider the following statements:

Deposit

Insurance and Credit Guarantee

reference

25

Q.59)

‘पर्ाय िरि प्रदशय ि सूचकाों क’ (The Environmental

Q.61)

Performance Index - EPI) के सोंदर्य में

कीनिए:

निम्ननलखित कथिमों पर निचार कीनिए: 1.

1.

पर्ायिरि प्रदशय ि सूचकाोंक, 2018 िे र्ह पार्ा 2.

नलए प्रमुि पर्ाय िरिीर् ितरा है ।

3.

र्ह इरािदी डॉखिि का पर्ाय िास है , निसे

‘र्े ल एिों कमलोंनबर्ा निश्वनिद्यालर्मों’ (Yale and

‘आई.र्ू .सी.एि.’

Columbia Universities) के शमधकतायओों

सोंकटग्रस्त

के द्वारा निनमयत की िाती है ।

सूचीबद्ध नकर्ा गर्ा है ।

खस्वट् ज़रलैंड धारिीर्ता के मामले में निश्व का

उपर्ुयक्त कथिमों में से कौि-सा/से सही है /हैं ? (a)

केिल 1

(b)

केिल 2 और 3

(c)

केिल 1 और 3

(d)

1, 2 और 3

(IUCN)

के

(Endangered)

के

अोंतगय त रूप

में

उपर्ुयक्त कथिमों में से कौि-सा सही है /हैं ?

िेतृत्व करता है ।

Q.62) Q.60)

र्ह ‘मॉन्टर े क्स ररकॉर् ड’ (The Montreax Record) में सूचीबद्ध है ।

र्ह नद्विानषयक ररपमटय ‘निश्व आनथयक मोंच’ (The World Economic Forum) के सहर्मग से

3.

िलबािा द्वीप इस झील में खस्थत है , िम एक पक्षी अभ्यारण्य है ।

है , नक िार्ु गु िित्ता, सािय िनिक स्वास्थ्य के

2.

नचल्का झील के सोंदर्य में निम्ननलखित कथिमों पर निचार

(a)

केिल 1

(b)

केिल 1 और 3

(c)

केिल 2 और 3

(d)

1, 2 और 3

‘अिुसूनचत िानत एिों अिुसूनचत िििानत (अत्याचार

'डी.एि.ए. बारकमनडों ग' (DNA Barcoding) के सोंदर्य

नििारि) अनधनिर्म, 1989’ [The Scheduled

में निम्ननलखित कथिमों पर निचार कीनिए:

Castes and Scheduled Tribes (Prevention of

1.

डी.एि.ए. बारकमनडों ग, नकसी र्ी िीि के छमटे -

Atrocities) Act, 1989] के सोंदर्य में निम्ननलखित

से ऊतक के िमूिे से डी.एि.ए. अिुक्रम

कथिमों पर निचार कीनिए:

निकालिे पर आधाररत, नकसी र्ी प्रिानत की

1.

त्वररत पहचाि की एक िई निनध है । 2.

िििानत के सदस्यमों के निरुद्ध अत्याचार के

डी.एि.ए. बारकमनडों ग एक िीि के सर्ी िीिि इनतहास चरिमों कम सखम्मनलत करिे के माध्यम से प्रिानतर्मों का निदाि करिे की क्षमता का

अपराधमों के कृत्यमों की रमकथाम करिा है । 2.

उपर्ुयक्त कथिमों में से कौि-सा/से सही है /हैं ?

र्ह अनर्र्ु क्तमों के नलए अपराधमों की सुििाई हे तु निशे ष न्यार्ालर्मों का प्रािधाि करता है ।

निस्तार करके पहचाि करिे में सहार्ता करती है ।

इसका उद्दे श्य अिुसूनचत िानत और अिुसूनचत

उपर्ुयक्त कथिमों में से कौि-सा/से सही है /हैं ? (a)

केिल 1

(a)

केिल 1

(b)

केिल 2

(b)

केिल 2

(c)

1 और 2 दमिमों

(c)

1 और 2 दमिमों

(d)

ि तम 1, ि ही 2

(d)

ि तम 1, ि ही 2

RAUSIAS-TC19E1007

26

Q.59) Consider

the

regarding

following the

statements

Environmental

Performance Index (EPI): 1.

The

2018

Environmental

Q.61) With reference to the Chilika Lake, consider the following statements: 1.

lake which is a Bird sanctuary.

Performance Index (EPI) finds that air

quality

is

environmental

the

threat

leading

2.

It is listed in the Montreax record.

public

3.

It is home to the Irrawady Dolphin

to

health. 2.

which

The biennial report is produced by researchers at Yale and Columbia Universities in collaboration with

3.

Nalabana island is located in this

is

listed

Which of the statements given above is/are correct?

Switzerland leads the world in

(a)

1 only

(b)

1 and 3 only

(c)

2 and 3 only

(d)

1, 2 and 3

Which of the statements given above is/are correct? (a)

1 only

(b)

2 and 3 only

(c)

1 and 3 only

(d)

1, 2 and 3

endangered

under IUCN.

the World Economic Forum. sustainability.

as

Q.62) Consider the following statements about the Scheduled Castes and Scheduled Tribes (Prevention of Atrocities) Act,

Q.60) Consider

the

following

statements

regarding ‘DNA barcoding’: 1.

DNA barcoding is a new method

1989: 1.

of offences of atrocities against the

for the quick identification of any species, based on extracting a DNA

members of the Scheduled Castes

sequence

and the Scheduled Tribes.

from

a

tiny

tissue

sample of any organism. 2.

It aims to prevent the commission

DNA

barcoding

identification by ability

to

assists expanding

diagnose

species

2. in

It provides for Special Courts for the

the

trial

of

offences

for

the

accused.

by

including all life history stages of

Which of the statements given above

an organism.

is/are correct?

Which of the statements given above is/are correct? (a)

1 only

(b)

2 only

(c)

Both 1 and 2

(d)

Neither 1 nor 2

RAUSIAS-TC19E1007

(a)

1 only

(b)

2 only

(c)

Both 1 and 2

(d)

Neither 1 nor 2

27

Q.63)

र्ारत में धि प्रेषि के सोंदर्य में निम्ननलखित कथिमों पर

Q.66)

तीि श्रेनिर्मों, र्था ‘खिटकर’ (Whitaker’s), ‘सहगल’

निचार कीनिए: 1.

2.

निम्ननलखित में से नकस प्रिानत कम अर्ी हाल ही में

र्ारत में धि प्रेषिमों में लमगमों के अन्य

(Sahgal’s) तथा ‘दनक्षिी टरमाइट’ (Southern

अथयव्यिस्थाओों में केिल अस्थार्ी आिागमि से

Termite) में उप-निर्ानित नकर्ा गर्ा है?

उत्पि हमिे िाले र्ुगताि प्राखप्त शानमल हैं, ि

(a)

मेंढक (Toad)

नक लमगमों के स्थार्ी आिागमि से।

(b)

नछपकली (Gecko)

धि प्रेषिमों कम र्ुगताि सोंतुलि के चालू िाते के

(c)

सपय (Snake)

अोंतगयत शानमल नकर्ा िाता है ।

(d)

नततली (Butterfly)

उपर्ुयक्त कथिमों में से कौि-सा/से सही है /हैं ?

Q.64)

Q.67)

केिल 1

(b)

केिल 2

(c)

1 और 2 दमिमों

1.

केंद्रीर् गृ हमोंत्ी इस निकार् के अध्यक्ष हमते हैं ।

(d)

ि तम 1, ि ही 2

2.

केंद्र शानसत प्रदे शमों के

निचार कीनिए:

निकास में िृ खद्ध करिा गृ ह मोंत्ालर् के नलए

निम्ननलखित में से नकसके सोंदर्य में 'STA-1' पद कर्ी(a)

अनििार्य नकर्ा गर्ा है । उपर्ुयक्त कथिमों में से कौि-सा/से सही है /हैं ?

सोंर्ुक्त राज्य अमेररका के साथ सोंिेदिशील प्रौद्यमनगनकर्मों का व्यापार

(b)

‘निश्व व्यापार सोंगठि’ (The World Trade Organisation – WTO) का नििाद निपटाि तों त्

(c)

इज़रार्ल की नमसाइल शील्ड प्रौद्यमनगकी

(d)

नब्रक्स (BRICS) मुद्रा व्यापार प्रिाली

‘बार्ममाकयसय’ (Biomarkers) के सोंदर्य में निम्ननलखित

Q.68)

(a)

केिल 1

(b)

केिल 2

(c)

1 और 2 दमिमों

(d)

ि तम 1, ि ही 2

‘आकाों शी निला कार्यक्रम’ के सोंदर्य में निम्ननलखित कथिमों पर निचार कीनिए: 1.

2.

बार्ममाकयर कम र्ा तम शरीर में पार्ा िा सकता

प्रत्ये क निले के नलए अनतररक्त सनचि/सोंर्ुक्त सनचि रैं क का एक केंद्रीर् प्रर्ारी अनधकारी

कथिमों पर निचार कीनिए: 1.

प्रशासकमों/राज्य

सरकार के साथ नमलकर द्वीप समूहमों के समग्र

कर्ी समाचारमों में नदिाई दे ता है ?

Q.65)

द्वीप निकास एिें सी के सोंदर्य में निम्ननलखित कथिमों पर

(a)

िानमत नकर्ा गर्ा है । 2.

र्ह कार्य क्रम औद्यमनगक निकास पर बल दे िे

है अथिा इसे बाह्य रूप से िीि में प्रनिष्ठ नकर्ा

के साथ आनथयक निकास पर ध्याि केंनद्रत

िा सकता है ।

करता है ।

एक आदशय बार्ममाकयर सर्ी नलों गमों और िातीर्

3.

से स्थािक्रनमत (ranked) नकए िाते हैं ।

समुदार्मों में सुसोंगत हमिा चानहए। उपर्ुयक्त कथिमों में से कौि-सा/से सही है /हैं ?

निले समर्-समर् पर डे ल्टा रैं नकोंग के माध्यम

उपर्ुयक्त कथिमों में से कौि-सा/से सही है /हैं ?

(a)

केिल 1

(a)

केिल 1 और 2

(b)

केिल 2

(b)

केिल 2 और 3

(c)

1 और 2 दमिमों

(c)

केिल 1 और 3

(d)

ि तम 1, ि ही 2

(d)

1, 2 और 3

RAUSIAS-TC19E1007

28

Q.63) Consider the following statements related to Remittances into India: 1.

2.

The Remittances into India includes the payment arising only from temporary movement of people to other economies and not from the permanent movement of people. The Remittances are accounted under the Current Account of the Balance of Payments (BoP).

Which of the statements given above is/are correct? (a)

1 only

(b)

2 only

(c)

Both 1 and 2

(d)

Neither 1 nor 2

Q.64) The term ‘STA-1’ is sometimes in news in the context of which of the following? (a)

Trade of sensitive with United States

technologies

(b)

Dispute settlement mechanism of the World Trade Organisation

(c)

Israeli Missile Shield technology

(d)

BRICS currency trading system

Q.65) With reference to Biomarkers, consider the following statements: 1.

2.

A biomarker can either be found existing in the body or can be introduced externally into an organism. An ideal biomarker should be consistent across gender and ethnic communities.

Which of the statements given above is/are correct?

Q.66) Which of the following specie has been recently sub divided into three categories namely Whitaker’s, Sahgal’s and Southern termite? (a)

Toad

(b)

Gecko

(c)

Snake

(d)

Butterfly

Q.67) Consider the following statements about the Island Development Agency: 1.

The Union Minister of Home Affairs is the Chairman of the body.

2.

The Ministry of Home Affairs has been mandated to steer the Holistic Development of Islands programme, along with UT/state governments.

Which of the statements given above is/are correct? (a)

1 only

(b)

2 only

(c)

Both 1 and 2

(d)

Neither 1 nor 2

Q.68) Consider the following statements about the ‘Aspirational District Programme’? 1.

For each district, a Central Prabhari Officer of the rank of Additional Secretary/Joint Secretary has been nominated.

2.

The programme focusses on economic development, with emphasis on industrial development.

3.

Districts are periodically ranked by Delta Rankings.

(a)

1 only

Which of the statements given above are correct?

(b)

2 only

(a)

1 and 2 only

(c)

Both 1 and 2

(b)

2 and 3 only

(d)

Neither 1 nor 2

(c)

1 and 3 only

(d)

1, 2 and 3

RAUSIAS-TC19E1007

29

Q.69)

िि धि र्मििा के सोंदर्य में निम्ननलखित कथिमों पर

Q.71)

निचार कीनिए: 1.

2.

5G सोंचार प्रौद्यमनगकी के सोंदर्य में निम्ननलखित कथिमों पर निचार कीनिए:

पर्ायिरि, िि एिों िलिार्ु पररितय ि मोंत्ालर्

1.

र्मििा के कार्ाय न्वर्ि के नलए िमडल मोंत्ालर्

दू रसोंचार मोंत्ालर् द्वारा एक उच्च स्तरीर् मोंच

है ।

का गठि नकर्ा गर्ा था।

इस र्मििा का लक्ष्य स्थािीर् स्तर पर उपलब्ध

2.

निकार् ितय माि में आई.टी.र्ू . की

हस्तनशल्प के मूल्सोंिधय ि कम प्रमत्ाहि प्रदाि

आिश्यकताओों के आधार पर 5G िेटिकय निकनसत कर रहा है ।

प्रत्ये क िि धि निकास केंद्र में 10 स्वर्ों

3.

सहार्ता समूह सोंलग्न हमोंगे।

(a)

केिल 1 और 2

(b)

केिल 2 और 3

(c)

केिल 1 और 3

(d)

1, 2 और 3

प्रौद्यमनगकी श्रेनिर्ाों हैं । उपर्ुयक्त कथिमों में से कौि-सा/से सही है /हैं ?

लघु ििमत्पाद र्मििा के नलए न्यूितम समथयि मूल् (MSP) के सोंदर्य में निम्ननलखित कथिमों पर निचार कीनिए: 1.

लघु िि उपि के नलए न्यूितम समथयि मूल् कृनष लागत और मूल् आर्मग (CACP) द्वारा

Q.72)

(a)

केिल 1 और 2

(b)

केिल 2 और 3

(c)

केिल 1 और 3

(d)

1, 2 और 3

‘दीिदर्ाल स्पशय र्मििा’ के सोंदर्य में निम्ननलखित कथिमों पर निचार कीनिए: 1.

लघु ििमत्पाद की िरीद का निस्तार क्षेत्

2.

3.

इस र्मििा का लक्ष्य दे श में डाक नटकट सोंग्रह कम प्रमत्ाहि दे िा है ।

सोंनिधाि की सूची-V में सूचीबद्ध क्षेत्मों के नलए सीनमत है ।

र्ह सोंचार मोंत्ालर् द्वारा शु रू नकर्ा गर्ा कार्यक्रम है ।

घमनषत नकर्ा िाता है । 2.

eMBB, mMTC तथा uRLLC 5G तकिीक के नलए 3GPP द्वारा प्रस्तानित तीि मुख्य

उपर्ुयक्त कथिमों में से कौि-सा/से सही है /हैं ?

Q.70)

3GPP, एक उद्यमग चानलत मािकीकरि

गै र-लकडी िि उत्पादमों एिों कौशल आधाररत

करिा है । 3.

दू रसोंचार मोंत्ालर् के सनचि की अध्यक्षता में

3.

र्ह कार्यक्रम डाक नटकट सोंग्रह में निशे ष रूनच

कुल 49 लघु ििमत्पाद इस र्मििा के अोंतगय त

दशाय िे िाले निश्वनिद्यालर् के छात्मों छात्िृ नत्त

आते हैं ।

सखम्मनलत करता है ।

उपर्ुयक्त कथिमों में से कौि-सा/से गलत है /हैं ?

उपर्ुयक्त कथिमों में से कौि-सा/से सही है /हैं ?

(a)

केिल 1 और 2

(a)

केिल 1 और 2

(b)

केिल 2 और 3

(b)

केिल 2 और 3

(c)

केिल 1 और 3

(c)

1 और 3

(d)

1, 2 और 3

(d)

1, 2 और 3

RAUSIAS-TC19E1007

30

Q.69) Consider the following statements about the Van Dhan Yojana: 1.

2.

The

Ministry

Q.71) Consider the following statements about the 5G Communication Technology:

of

Environment,

1.

Forest and Climate Change is the

by

nodal

Telecommunications,

ministry

for

the

the

Ministry

of

headed

by

implementation of the Scheme.

the Secretary, the Department of

The Scheme aims to promote value

Telecommunications.

addition of locally available non-

2.

timber forest produce and skill

have 10 Self Help Groups (SHGs) attached to it.

3.

2 and 3 only

(c)

1 and 3 only

(d)

1, 2 and 3

driven

eMBB, mMTC and uRLLC are the three main technology categories proposed by 3GPP for the 5G

correct? (b)

industry

the ITU requirements.

Which of the statements given above are 1 and 2 only

an

developing 5G networks based on

Each Van Dhan Vikas Kendra will

(a)

‘3GPP’,

standardisation body, is currently

based handicrafts. 3.

A high level forum was constituted

technology. Which of the statements given above are correct?

Q.70) Consider the following statements about MSP (Minimum Support Price) for the

(a)

1 and 2 only

(b)

2 and 3 only

(c)

1 and 3 only

(d)

1, 2 and 3

Minor Forest Produce Scheme: 1.

The MSP for the minor forest

Q.72) Consider the following statements about

produce (MFP) is declared by CACP

the ‘Deen Dayal Sparsh Yojana’:

(The Commission for Agricultural

1.

Ministry of Communications.

Costs and Prices). 2.

The

area

procurement

of of

coverage minor

for

2.

forest

produce is limited to Schedule-V 3.

The programme is launched by the The

Scheme

aims

to

promote

philately in the country. 3.

The

programme

will

involve

a

areas listed in the Constitution.

scholarship to university students

Total 49 minor forest produce are

showing

covered under the Scheme.

philately.

special

interest

in

Which of the statements given above are

Which of the statements given above are

incorrect?

correct?

(a)

1 and 2 only

(a)

1 and 2 only

(b)

2 and 3 only

(b)

2 and 3 only

(c)

1 and 3 only

(c)

1 and 3 only

(d)

1, 2 and 3

(d)

1, 2 and 3

RAUSIAS-TC19E1007

31

Q.73)

कपडा क्षेत् के नलए ‘समथय र्मििा’के सोंदर्य में

Q.75)

कथिमों पर निचार कीनिए:

निम्ननलखित कथिमों पर निचार कीनिए: 1.

िाि सुरक्षा महानिदे शालर् के सोंदर्य में निम्ननलखित 1.

इस र्मििा का लक्ष्य कपडा क्षेत् की सोंपूिय

के सोंनिधाि की अिुसूची-VII की समिती सूची

मूल् श्रृोंिला के नलए कपडा क्षेत् में कौशल

2.

का र्ाग है ।

अोंतराल कम पूरा करिा है ।

2.

र्ह िाि मोंत्ालर् के अधीि कार्य करता है ।

इस र्मििा में केिल सोंगनठत कपडा क्षेत्

3.

र्ह िदािमों (कमर्ला, धातु एिों कच्चे ते ल की िदािमों) में कार्यरत व्यखक्तर्मों की व्यािसानर्क

शानमल है ।

सुरक्षा, स्वास्थ्य एिों कल्ाि से सोंबोंनधत मामलमों के नलए निर्ामक अनर्करि है ।

उपर्ुयक्त कथिमों में से कौि-सा/से सही है /हैं ?

Q.74)

(a)

केिल 1

(b)

केिल 2

(c)

1 और 2 दमिमों

(d)

ि तम 1, ि ही 2

‘इों नडर्ा साइि (India Size)’ के सोंदर्य में निम्ननलखित कथिमों पर निचार कीनिए: 1.

र्ह कपडा मोंत्ालर् के अधीि राष्ट्रीर् आकार

उपर्ुयक्त कथिमों में से कौि-सा/से सही है /हैं ?

Q.76)

(a)

केिल 1 और 3

(b)

केिल 2 और 3

(c)

केिल 3

(d)

1, 2 और 3

‘निनित

1.

इसमें नत्आर्ामी बॉडी स्कैिर के उपर्मग से

माििरूपी माप नलर्ा िाएगा। 4.

2.

अिुबोंध के आधार पर कार्य में सोंलग्न हमता है । 3.

केिल 2, 3 और 4

(c)

केिल 1, 2 और 4

(d)

1, 2, 3 और 4

RAUSIAS-TC19E1007

एक निनित अिनध के कमयचारी के कार्य, िे ति, र्त्ते एिों अन्य लार् स्थार्ी कमयचारी से कम िहीों हमोंगे।

4.

निनित अिनध का कमयचारी स्थार्ी सेिा के नलए उपलब्ध सर्ी िै धानिक लार्मों के नलए पात् हमगा, िम नक उसके द्वारा प्रदत्त सेिा की अिनध के अिुसार समाि रूप से उपलब्ध है , र्ले ही उसके रमिगार की अिनध िै धानिक में

उपर्ुयक्त कथिमों में से कौि-सा/से सही है /हैं ?

(b)

निनित अिनध के रमिगार का कमयचारी िह हमता है , िम एक निनित अिनध के नलए नलखित

नक्रर्ान्वर्ि करें गा।

केिल 1, 2, और 3

निर्मक्ता अपिे औद्यमनगक प्रनतष्ठाि में मौिू दा

है ।

आई.आई.टी. का सोंघ इस पररर्मििा का

(a)

Term

रमिगार के रूप में पररिनतय त िहीों कर सकता

र्ारत के 6 अलग-अलग क्षेत्मों में 18 से 65 िषय की आर्ु के 25,000 व्यखक्तर्मों के नलए

(Fixed

स्थार्ी कमयचाररर्मों के पदमों कम निनित अिनध के

उपर्मग करता है । 3.

रमिगार

निचार कीनिए:

ितय माि में र्ारतीर् पररधाि उद्यमग अन्य दे शमों के आकार (Size) चाटय के सोंस्करिमों का

अिनध

Employment)’ के सोंदर्य में निम्ननलखित कथिमों पर

मापि (Sizing) सिे क्षि है । 2.

िदािमों में श्रम एिों सुरक्षा का निनिर्मि र्ारत

आिश्यक रमिगार की र्मग्यता अिनध तक निस्तृत िहीों हम। उपर्ुयक्त कथिमों में से कौि-सा/से सही है /हैं ? (a)

केिल 1, 2 और 3

(b)

केिल 1, 2 और 4

(c)

केिल 2, 3 और 4

(d)

1, 2, 3 और 4

32

Q.73) Consider the following statements about the ‘SAMARTH Scheme’ for the textile sector: 1.

The Scheme aims to meet the skill gap in the textile sector for entire value chain of the textile sector.

2.

The

Scheme

covers

only

the

organised textile sector. Which of the statements given above is/are correct? (a)

1 only

(b)

2 only

(c)

Both 1 and 2

(d)

Neither 1 nor 2

Q.74) Consider the following statements about ‘INDIA SIZE’: 1.

It is a national sizing survey, under the Ministry of Textiles.

2.

Currently,

the

Indian

apparel

industry uses tweaked versions of other countries’ size charts. 3.

Anthropomorphic

measurements

will be taken for 25,000 persons aged from 18 years to 65 years in 6 different regions of India, using 3D body scanners. 4.

Consortium of IITs (the Indian Institute

of

Technology)

will

implement the Project. Which of the statements given above are correct? (a)

1, 2, and 3 only

(b)

2, 3 and 4 only

(c)

1, 2 and 4 only

(d)

1, 2, 3 and 4

RAUSIAS-TC19E1007

Q.75) Consider the following statements about the Directorate General of Mines Safety: 1. Regulation of labour and safety in mines is a part of the Concurrent List of Schedule VII to the Constitution of India. 2. It functions under the Ministry of Mines. 3. It is a regulatory agency for matters pertaining to occupational safety, health and welfare of persons employed in mines (coal, metalliferous and oil mines) Which of the statements given above is/are correct? (a) 1 and 3 only (b) 2 and 3 only (c) 3 only (d) 1, 2 and 3 Q.76) Consider the following statements about ‘Fixed Term Employment’: 1. The employers cannot convert posts of permanent workmen existing in their industrial establishments as Fixed Term Employment. 2. Fixed Term Employment workman is a workman who has been engaged on the basis of a written contract for a fixed term. 3. The hours of work, wages, allowances and other benefits of a Fixed Term Employee shall not be less than that of a permanent workman. 4. A Fixed Term Employee shall be eligible for all statutory benefits available to a permanent workman proportionately, according to the period of service rendered by him, even if his period of employment does not extend to the qualifying period of employment required in the statute. Which of the statements given above are correct? (a) 1, 2 and 3 only (b) 1, 2 and 4 only (c) 2, 3 and 4 only (d) 1, 2, 3 and 4

33

Q.77)

‘अटल बीनमत व्याखक्त कल्ाि र्मििा’ के सोंदर्य में

Q.79)

निम्ननलखित कथिमों पर निचार कीनिए:

पेटरमनलर्म, रसार्ि एिों पेटरमरसार्ि नििेश क्षेत्मों के सोंदर्य में निम्ननलखित कथिमों पर निचार कीनिए:

1.

र्ह नित्त मोंत्ालर् द्वारा शु रू की गई र्मििा है ।

2.

र्ह र्मििा प्रमनिडें ट निनध िातमों में र्मगदाि

िीनत पेटरमनलर्म एिों प्राकृनतक गै स मोंत्ालर्

करिे के बाद बे रमिगारी में रूपाोंतररत सर्ी

द्वारा लागू की िाती है ।

1.

लमगमों कम बे रमिगारी लार् प्रदाि करती है ।

2.

पेटरमनलर्म, रसार्ि एिों पेटरमरसार्ि नििे श क्षेत्मों कम गु िरात, महाराष्ट्र, आों ध्र प्रदे श और

उपर्ुयक्त कथिमों में से कौि-सा/से सही है /हैं ?

Q.78)

पेटरमनलर्म, रसार्ि एिों पेटरमरसार्ि नििे श क्षेत्मों

(a)

केिल 1

तनमलिाडु में स्थानपत नकर्ा िा रहा है ।

(b)

केिल 2

उपर्ुयक्त कथिमों में से कौि-सा/से सही है /हैं ?

(c)

1 और 2 दमिमों

(a)

केिल 1

(d)

ि तम 1, ि ही 2

(b)

केिल 2

(c)

1 और 2 दमिमों

(d)

ि तम 1, ि ही 2

निम्ननलखित र्ु ग्मों पर निचार कीनिए: प्रस्तावित बंदरगाह

राज्य

1.

एिार्म

:

किाय टक

2.

िाधिि

:

महाराष्ट्र

3.

तािपुर

:

Q.80)

माल एिों सेिा कर (िी.एस.टी.) िेटिकय के सोंदर्य में निम्ननलखित कथिमों पर निचार कीनिए: 1.

र्ह एक गै र-लार्कारी कोंपिी के नलए है ।

2.

मोंनत्मोंडल िे इसे 50% केंद्र के स्वानमत्व तथा

पनिम बों गाल

4.

नसरकाषी

:

आों ध्र प्रदे श

5.

बे लेकेरी

:

ओनडशा

उपर्ुयक्त र्ु ग्मों में से कौि-सा/से सही सुमेनलत है/हैं?

50% राज्यमों के स्वानमत्व के साथ 100% सािय िनिक कोंपिी बिािे का निियर् नलर्ा है । उपर्ुयक्त कथिमों में से कौि-सा/से सही है /हैं ?

(a)

केिल 1, 4 और 5

(a)

केिल 1

(b)

केिल 2, 3 और 5

(b)

केिल 2

(c)

केिल 2 और 3

(c)

1 और 2 दमिमों

(d)

केिल 1 और 5

(d)

ि तम 1, ि ही 2

RAUSIAS-TC19E1007

34

Q.77) Consider the following statements about

Q.79) Consider the following statements about

the ‘Atal Bimit Vyakti Kalyan Yojana’:

the

1.

It is a Scheme launched by the

Petrochemical

Ministry of Finance.

(PCPIRs):

2.

The

Scheme

unemployment persons after

provides

benefits

rendered

to

1.

all

to

their

Provident Fund Accounts.

2.

(b)

2 only

(c)

Both 1 and 2

(d)

Neither 1 nor 2

Regions

The PCPIR Policy is implemented

PCPIRs are being established in Gujarat,

Maharashtra,

Andhra

Pradesh and Tamil Nadu.

is/are correct? 1 only

Investment

and

Natural Gas.

Which of the statements given above

(a)

Chemicals

by the Ministry of Petroleum and

unemployed

contributing

Petroleum,

Which of the statements given above is/are correct?

Q.78) Consider the following pairs:

(a)

1 only

(b)

2 only

(c)

Both 1 and 2

(d)

Neither 1 nor 2

State

Proposed Ports

Q.80) Consider the following statements about

1.

Enayam

:

Karnataka

the GST Network:

2.

Vadhavan

:

Maharashtra

1.

It is a not-for-profit company.

3.

Tajpur

:

West Bengal

2.

The Cabinet has decided to make

4.

Sirkazhi

:

Andhra

it a, 100% public company with

Pradesh

50% centre’s ownership and 50%

Odisha

states ownership.

5.

Belekeri

:

Which of the pairs given above are

Which of the statements given above

correctly matched?

is/are correct?

(a)

1, 4 and 5 only

(a)

1 only

(b)

2, 3 and 5 only

(b)

2 only

(c)

2 and 3 only

(c)

Both 1 and 2

(d)

1 and 5 only

(d)

Neither 1 nor 2

RAUSIAS-TC19E1007

35

Q.81)

‘राष्ट्रीर् स्वास्थ्य प्रानधकरि’ के सोंदर्य में निम्ननलखित

Q.83)

कथिमों पर निचार कीनिए: 1.

सोंदर्य में निम्ननलखित कथिमों पर निचार कीनिए:

राष्ट्रीर् स्वास्थ्य प्रानधकरि के गिनििंग बमडय की अध्यक्षता प्रधािमोंत्ी द्वारा की िाएगी।

2.

3.

‘मातृ , िििात नशशु एिों बाल स्वास्थ्य की र्ागीदारी’के

र्ह प्रधािमोंत्ी िि आरमग्र् र्मििा के सुचारू

1.

र्ह एक अोंतर-सरकारी निकार् है ।

2.

मातृ, िििात नशशु एिों बाल स्वास्थ्य की

कार्ाय न्वर्ि के नलए उत्तरदार्ी हमगा।

र्ागीदारी का सनचिालर् निश्व स्वास्थ्य सोंगठि

राष्ट्रीर् स्वास्थ्य प्रानधकरि के गिनििंग बमडय में

के द्वारा आर्मनित एिों प्रशानसत है ।

राज्यमों का प्रनतनिनधत्व में र्ी नकर्ा िाएगा।

3.

उपर्ुयक्त कथिमों में से कौि-सा/से सही है /हैं ?

Q.82)

(a)

केिल 1 और 2

(b)

केिल 2 और 3

(c)

केिल 1 और 3

(d)

1, 2 और 3

पाटय िसय फमरम मातृ, िििात नशशु एिों बाल स्वास्थ्य की र्ागीदारी के नलए एक िै नश्वक मोंच के रूप में कार्य करता है ।

उपर्ुयक्त कथिमों में से कौि-सा/से सही है /हैं ? (a)

केिल 1 और 2

(b)

केिल 2 और 3

एच.आई.िी. और एड् स (रमकथाम और निर्ोंत्ि)

(c)

केिल 1 और 3

अनधनिर्म, 2017 के सोंदर्य में निम्ननलखित कथिमों पर

(d)

1, 2 और 3

निचार कीनिए: 1.

इस अनधनिर्म के अिुसार, सोंरनक्षत व्यखक्त िे व्यखक्त हैं , िम केिल एच.आई.िी. पॉनिनटि हैं ।

2.

एच.आई.िी. और एड् स से पीनडत प्रत्येक व्यखक्त कम साझा घर से बाहर िहीों हमिे का

3.

सोंदर्य में निम्ननलखित कथिमों पर निचार कीनिए: 1.

र्ह स्कूल नशक्षा पर राज्यमों और केंद्र शानसत प्रदे शमों के प्रदशय ि की ग्रे नडों ग करिे के नलए िीनत

नकसी र्ी व्यखक्त कम नकसी र्ी पररखस्थनत में

आर्मग की एक पहल है ।

नलए नििश िहीों नकर्ा िाएगा।

2.

इस सूचकाोंक के नलए डे टा का तृ तीर् पक्ष सत्यापि र्ू निसेफ के द्वारा नकर्ा िाएगा।

राज्य सरकारमों कम लमकपाल निर्ु क्त करिा हमगा, निसे इस अनधनिर्म के प्रािधािमों के

5.

हाल ही में शु रू नकए गए परफॉमेंस ग्रे नडों ग इों डेक्स के

अनधकार है ।

अपिी एच.आई.िी. खस्थनत कम प्रकट करिे के

4.

Q.84)

3.

ग्रे नडों ग पररिाममों एिों अनधशासि प्रनक्रर्ाओों कम

उल्लोंघि के नलए नशकार्त की िा सकती है ।

ध्याि में रिा िाएगा तथा इसमें 70 सोंकेतक

र्ह अनधनिर्म रमिगार प्राप्त करिे के नलए पूिय

हमोंगे।

एच.आई.िी. परीक्षि कम प्रनतबों नधत करता है । उपर्ुयक्त कथिमों में से कौि-सा/से सही है /हैं ?

उपर्ुयक्त कथिमों में से कौि-सा/से सही है /हैं ? (a)

केिल 1 और 2

केिल 2, 4 और 5

(b)

केिल 2 और 3

(c)

केिल 1, 3 और 4

(c)

केिल 1 और 3

(d)

1, 2, 3, 4 और 5

(d)

1, 2 और 3

(a)

केिल 1, 2, 3 और 4

(b)

RAUSIAS-TC19E1007

36

Q.81) Consider the following statement about the ‘National Health Authority’: 1.

2.

3.

The Governing Board of the National Health Authority will be chaired by the Prime Minister. It will be responsible for the smooth implementation of PM Jan Aarogya Yojana. The states shall also be represented in the Governing Board of the National Health Authority.

Q.83) Consider the following statements about ‘Partnership of Maternal, Newborn and Child Health (PMNCH): 1.

It is an inter-governmental body.

2.

The secretariat of the PMNCH is hosted and administered by the World Health Organisation (WHO).

3.

The Partners’ Forum serves as a global platform for the PMNCH.

Which of the statements given above are correct?

Which of the statements given above are

(a)

1 and 2 only

correct?

(b)

2 and 3 only

(a)

1 and 2 only

(c)

1 and 3 only

(d)

1, 2 and 3

(b)

2 and 3 only

(c)

1 and 3 only

(d)

1, 2 and 3

Q.82) Consider the following statements about the HIV and AIDS (Prevention and Control) Act, 2017: 1.

2.

3.

4.

5.

The protected persons, according to the Act, are persons who are HIV positive only. Every person suffering from HIV and AIDS has a right not to be excluded from the shared household. No person shall be compelled to disclose his/her HIV status under any circumstances. State governments have to appoint an Ombudsman to whom a complaint can be made for the violations of the provisions of this Act.

Q.84) Consider the following statements about the

1.

Index,

It is an initiative of the NITI Aayog for grading States and UTs on their

performance

on

school

education. 2.

Third party validation of data for the Index will be done by UNICEF.

3.

The grading will take into account ‘outcomes’ processes

and and

‘governance’ will

have

70

indicators. Which of the statements given above are correct?

(a)

1, 2, 3 and 4 only

(b)

2, 4 and 5 only

(a)

1 and 2 only

(c)

1, 3 and 4 only

(b)

2 and 3 only

(d)

1, 2, 3, 4 and 5

(c)

1 and 3 only

(d)

1, 2 and 3

RAUSIAS-TC19E1007

Grading

launched recently:

The act prohibits HIV testing as a pre-requisite for obtaining employment.

Which of the statements given above are correct?

Performance

37

Q.85)

प्रधािमोंत्ी आदशय ग्राम र्मििा के सोंदर्य में निम्ननलखित

Q.87)

निम्ननलखित कथिमों पर निचार कीनिए:

कथिमों पर निचार कीनिए: 1.

गिियमेंट-ई-माकेटप्लेस (िी.ई.एम. पमटय ल) के सोंदर्य में 1.

र्ह ग्रामीि निकास मोंत्ालर् द्वारा सोंचानलत एक

गए प्रमुि कदममों में से एक है ।

र्मििा है । 2.

2.

र्ह अिुसूनचत िानत की 50 प्रनतशत से अनधक

3.

गाों िमों पर केंनद्रत है ।

उपर्ुयक्त कथिमों में से कौि-सा/से गलत है /हैं ?

filling fund) का प्रािधाि करता है, िम नक गाों ि की निशे ष आिश्यकताओों, निन्हें मौिू दा र्मििाओों में से नकसी के अोंतगय त पूरा िहीों नकर्ा िा सकता है , कम पूरा करिे के नलए उपर्मग नकर्ा िाएगा। Q.88)

(a)

केिल 3

(b)

केिल 2 और 3

(c)

केिल 1 और 3

(d)

केिल 2

राष्ट्रीर् औषध मूल् निधाय रि प्रानधकरि के सोंदर्य में निम्ननलखित कथिमों पर निचार कीनिए:

(a)

केिल 1 और 2

(b)

केिल 2 और 3

1.

र्ह एक साों निनधक निकार् है ।

(c)

केिल 1 और 3

2.

र्ह आिश्यक िस्तु अनधनिर्म के अोंतगयत

(d)

1, 2 और 3

दिाओों का मूल्मों निधाय ररत करता है । 3.

Q.86)

र्ह िानिज्य एिों उद्यमग मोंत्ालर् के अधीि कार्य करता है ।

इस र्मििा में अोंतराल र्रि निनध (Gap

उपर्ुयक्त कथिमों में से कौि-सा/से सही है /हैं ?

इसका उपर्मग केिल केंद्र सरकार द्वारा नकर्ा िाता है ।

तथा 500 से अनधक की कुल ििसाँख्या िाले 3.

र्ह सािय िानिक िरीद सुधार के नलए उठाए

करता है ।

नमशि अोंत्यमदर् के सोंदर्य में निम्ननलखित कथिमों पर 4.

निचार कीनिए: 1.

5.

उपर्ुयक्त कथिमों में से कौि-सा/से सही है /हैं ?

सखम्मनलत है । र्ह ग्रामीि क्षेत्मों के निकास के नलए लनक्षत कार्यक्रममों का अनर्सरि सुनिनित करता है । 4.

इस र्मििा के अोंतगय त र्ौनतक अिसोंरचिा के निकास एिों मािि निकास के साथ आनथयक गनतनिनधर्मों के सोंचालि के नलए लगर्ग 5000 समूहमों में 50,000 ग्राम पोंचार्तमों का चर्ि नकर्ा गर्ा है ।

उपर्ुयक्त कथिमों में से कौि-सा/से सही है /हैं ? (a)

केिल 1, 2 और 3

(b)

र्ह स्वास्थ्य एिों पररिार कल्ाि मोंत्ालर् के अधीि कार्य करता है ।

इसमें पररिारमों का आधार रे िा सिे क्षि एिों समर्-समर् पर प्रगनत की निगरािी करिा

3.

र्ह गै र-अिुसूनचत दिाओों का मूल्मों की निगरािी करता है ।

ग्रामीि निकास मोंत्ालर् के द्वारा इसका नक्रर्ान्वर्ि नकर्ा िाएगा।

2.

र्ह अिुसूनचत दिाओों का मूल्मों निधाय ररत

Q.89)

(a)

केिल 1, 2 और 3

(b)

केिल 2, 3 और 4

(c)

केिल 3, 4 और 5

(d)

केिल 1, 3 और 5

निम्ननलखित कथिमों पर निचार कीनिए: 1.

मट्टिचे री महल चमल साम्राज्य द्वारा बिार्ा गर्ा था।

2.

र्ह तनमलिाडु में खस्थत है ।

उपर्ुयक्त कथिमों में से कौि-सा/से सही है /हैं ? (a)

केिल 1

केिल 1, 2 और 4

(b)

केिल 2

(c)

केिल 2, 3 और 4

(c)

1 और 2 दमिमों

(d)

1, 2, 3 और 4

(d)

ि तम 1, ि ही 2

RAUSIAS-TC19E1007

38

Q.85) Consider the following statements about the PM Adarsh Gram Yojana: 1.

It is a Scheme operated by the Ministry of Rural Development.

2.

It focuses on villages with SC population of more than 50 percent and a total population of more than 500.

3.

The Scheme provides for Gap Filling Fund, which will be utilised for meeting special requirements of the village, which cannot be met under any of the existing Schemes.

Which of the statements given above are correct? (a)

1 and 2 only

(b)

2 and 3 only

(c)

1 and 3 only

(d)

1, 2 and 3

Q.86) Consider the following statements about Mission Antyodaya: 1.

It will be implemented by the Ministry of Rural Development.

2.

It involves carrying out the baseline survey of households and monitor the progress periodically.

3.

4.

It will ensure convergence of programmers targeted towards development of rural areas. 50,000 Gram Panchayats have been selected in about 5,000 clusters to drive economic activities along with the development of physical infrastructure and human development under the Scheme.

Q.87) Consider the following statements about the Government-e-Marketplace (GeM) portal: 1.

It is one of the major steps for public procurement reform.

2.

It is only used by the Central government.

3.

It functions under the Ministry of Commerce and Industry.

Which of the statements given above is/are incorrect? (a)

3 only

(b)

2 and 3 only

(c)

1 and 3 only

(d)

2 only

Q.88) Consider the following statements about the National Pharmaceutical Pricing Authority (NPPA): 1.

It is a statutory body.

2.

It fixes the prices of drugs under the Essential Commodities Act.

3.

It fixes the prices of scheduled drugs.

4.

It also monitors the prices of nonscheduled drugs.

5.

It functions under the Ministry of Health and Family Welfare.

Which of the statements given above are correct? (a)

1, 2 and 3 only

(b)

2, 3 and 4 only

(c)

3, 4 and 5 only

(d)

1, 3 and 5 only

Q.89) Consider the following statements: 1.

Mattanchery Palace was built by the Chola Empire.

2.

It is located in Tamil Nadu.

Which of the statements given above are correct?

Which of the statements given above is/are correct?

(a)

1, 2 and 3 only

(a)

1 only

(b)

1, 2 and 4 only

(b)

2 only

(c)

2, 3 and 4 only

(c)

Both 1 and 2

(d)

1, 2, 3 and 4

(d)

Neither 1 nor 2

RAUSIAS-TC19E1007

39

Q.90)

निष्णुपुर मोंनदरमों के सोंदर्य में निम्ननलखित कथिमों पर

Q.93)

कौिसा/से राष्ट्रीर् उद्याि/िन्यिीि अर्र्ारण्य

निचार कीनिए: 1. 2.

निष्णुपुर के मोंनदर अनधकतर एक-रत्न प्रकार के

सखम्मनलत है /हैं ?

हैं ।

1.

िमकरे क राष्ट्रीर् उद्याि

र्े ईोंटमों से बिे हैं ।

2.

बालपकराम राष्ट्रीर् उद्याि

3.

नसिु िन्यिीि अर्र्ारण्य तु रा चमटी के आरनक्षत िि

उपर्ुयक्त कथिमों में से कौि-सा/से सही है /हैं ?

Q.91)

गारम पहाडी सोंरक्षि क्षेत् में निम्ननलखित में से

(a)

केिल 1

4.

(b)

केिल 2

िीचे नदए गए कूट का प्रर्मग कर सही उत्तर चु निए:

(c)

1 और 2 दमिमों

(a)

केिल 1

(d)

ि तम 1, ि ही 2

(b)

केिल 2 और 3

(c)

केिल 2, 3 और 4

(d)

1, 2, 3 और 4

हाल ही में ओनडशा के मुख्यमोंत्ी ििीि पटिार्क िे गु रु पद्मसोंर्ि की 19 फुट ऊोंची प्रनतमा का अिािरि नकर्ा है , इिके सोंदर्य में निम्ननलखित कथिमों पर निचार

Q.94)

1.

कीनिए: 1.

निम्ननलखित कथिमों पर निचार कीनिए: गों गा के मैदािी इलाकमों में उत्तरी काले

गु रु पद्मसोंर्ि नतब्बती बौद्ध धमय के सोंस्थापक

पॉनलशदार मृदर्ाोंड (NBPW) चरि की

थे। 2.

शु रुआत हुई थी।

उन्हें (गुरु पद्मसोंर्ि) कम दू सरे बु द्ध के रूप में

2.

र्ी िािा िाता है ।

Q.92)

केिल 1

(b)

केिल 2

(c)

1 और 2 दमिमों

(d)

ि तम 1, ि ही 2

निम्ननलखित में से कौि सा दे श '6 + 1 समूह' में शानमल है ? 1.

अफनगस्ताि

2.

र्ारत

3.

पानकस्ताि

4.

रूस

िीचे नदए गए कूट का प्रर्मग कर सही उत्तर चु निए: (a)

केिल 1 और 2

(b)

केिल 2 और 3

(c)

केिल 1, 2 और 4

(d)

1, 2, 3 और 4

RAUSIAS-TC19E1007

एि.बी.पी.डब्ल्यू. चरि िे र्ारत में तीसरे शहरीकरि की शु रुआत कम नचनित नकर्ा था।

उपर्ुयक्त कथिमों में से कौि-सा/से सही है /हैं ? (a)

पुराताखत्वक रूप से पाों चिीों शताब्दी ईसा पूिय में

उपर्ुयक्त कथिमों में से कौि-सा/से सही है /हैं ?

Q.95)

(a)

केिल 1

(b)

केिल 2

(c)

1 और 2 दमिमों

(d)

ि तम 1, ि ही 2

नसोंधु घाटी सभ्यता के सोंदर्य में निम्ननलखित कथिमों पर निचार कीनिए: 1.

र्हााँ उगाई िािे िाली मुख्य फसलें गे हाँ और िौ थी।

2.

र्ेड, बकररर्ाों और र्ैंस िै से पशु ओों कम पालतू बिा नलर्ा गर्ा था।

3.

घमडे का उपर्मग इस काल तक स्थानपत िहीों हुआ था।

उपर्ुयक्त कथिमों में से कौिसा/से सही है /हैं ? (a)

केिल 1

(b)

केिल 2

(c)

केिल 1 और 2

(d)

1, 2 और 3

40

Q.90) Consider the following statements about the Bishnupur temples: 1.

The temples at Bishnupur are mostly of Eka-Ratna type.

2.

They are built in bricks.

Q.93) The Garo Hills Conservation Area (GHCA) comprises which of the following National Parks/Wildlife Sanctuary? 1.

Nokrek National Park

2.

Balpakram National Park

Which of the statements given above

3.

Siju Wildlife Sanctuary

is/are correct?

4.

Reserved Forests of Tura Peak

(a)

1 only

(b)

2 only

Select the correct answer using the code given below:

(c)

Both 1 and 2

(d)

Neither 1 nor 2

(a)

1 only

(b)

2 and 3 only

(c)

2, 3 and 4 only

(d)

1, 2, 3 and 4

Q.91) Recently, Odisha Chief Minister Naveen Patnaik unveiled a 19-foot-high statue of Guru Padmasambhava. Consider the

Q.94) Consider the following statements: 1.

Archaeologically, the fifth century BC marks the beginning of the Northern Black Polished Ware (NBPW) phase in the Gangetic plains.

2.

The NBPW phase marked the beginning of the third urbanization in India.

following statements regarding this: 1.

Guru Padmasambhava was the founder of Tibetan Buddhism.

2.

He (Guru Padmasambhava) is also known as Second Buddha.

Which of the statements given above is/are correct? (a)

1 only

(b)

2 only

(c) (d)

Which of the statements given above is/are correct? (a)

1 only

(b)

2 only

Both 1 and 2

(c)

Both 1 and 2

Neither 1 nor 2

(d)

Neither 1 nor 2

Q.92) Which of the following countries are included in the ‘6 + 1’ Group’? 1.

Afghanistan

2.

India

3.

Pakistan

4.

Russia

Select the correct answer using the code

Q.95) Consider the following statements about the Indus Valley Civilization: 1.

Wheat and barley were the main crops grown.

2.

Animals like sheep, goats and buffaloes were domesticated.

3.

The use of horse is not yet firmly established.

given below:

Which of the statements given above is/are correct?

(a)

1 and 2 only

(a)

1 only

(b)

2 and 3 only

(b)

2 only

(c)

1, 2 and 4 only

(c)

1 and 2 only

(d)

1, 2, 3 and 4

(d)

1, 2 and 3

RAUSIAS-TC19E1007

41

Q.96) नसोंधु घाटी सभ्यता के सोंदर्य में निम्ननलखित कथिमों पर निचार कीनिए: 1.

Q.99) निम्ननलखित कथिमों पर निचार कीनिए: 1.

नबखम्बसार हर्यक िोंश से सम्बों नधत था तथा

कलश शिाधाि की प्रथा कर्ी-कर्ी कोंकालमों

उसिे िैिानहक सोंबोंधमों के माध्यम से अपिी

के र्ुग्मों के साथ लमथल में पाई गई है ।

खस्थनत सुदृढ बिाई थी

2.

र्हााँ सती प्रथा के नलए स्पष्ट् साक्ष्य मौिू द हैं ।

3.

हडप्पा की नलनप कम समझा िा चुका है ।

उपर्ुयक्त कथिमों में से कौि-सा/से सही है /हैं ?

2.

िह िधयमाि महािीर तथा गौतम बुद्ध का समकालीि था।

उपर्ुयक्त कथिमों में से कौि-सा/से सही है /हैं ?

(a)

केिल 1

(a)

केिल 1

(b)

केिल 2

(b)

केिल 2

(c)

केिल 1 और 2

(c)

1 और 2 दमिमों

(d)

1, 2 और 3

(d)

ि तम 1, ि ही 2

Q.97) ऋग िैनदक काल के सोंदर्य में निम्ननलखित कथिमों पर निचार कीनिए: 1. 2.

ऋग िैनदक आर्य पशु चारी लमग थे तथा

Q.100) नसकोंदर के आक्रमि के प्रर्ािमों के सोंदर्य में निम्ननलखित कथिमों पर निचार कीनिए: 1.

उिका मु ख्य पेशा पशु पालि था।

र्ह था नक इसिे मौर्ों के अधीि उत्तर र्ारत

उिका धि उिके मिेनशर्मों के रूप में

के राििीनतक एकीकरि का प्रमत्ानहत नकर्ा था।

आकनलत नकर्ा िाता था। उपर्ुयक्त कथिमों में से कौि-सा/से सही है /हैं ? (a)

केिल 1

(b)

केिल 2

(c)

1 और 2 दमिमों

(d)

ि तम 1, ि ही 2

Q.98) बौद्ध धमय के सोंदर्य में निम्ननलखित कथिमों पर निचार कीनिए: 1.

नसकोंदर के आक्रमि का तात्कानलक प्रर्ाि

सोंघ कम लमकताों नत्क मू ल्मों से पोंखक्तर्मों पर अनधशानसत नकर्ा गर्ा था तथा र्ह अपिे सदस्यमों के मध्य अिु शासि लागू करिे के

2.

नसकोंदर के आक्रमि िे र्ारत तथा र्ूिाि के मध्य प्रत्यक्ष सोंपकय का मागय प्रशस्त नकर्ा था।

3.

उसके िमले गए मागों तथा उसके िौसैनिक अन्वे षिमों िे र्ारत तथा पनिम एनशर्ा के मध्य व्यापर के नलए मौिू दा सुनिधाओों में िृखद्ध की थी।

उपर्ुयक्त कथिमों में से कौि-सा/से सही है /हैं ? (a)

केिल 1

(b)

केिल 2

(c)

केिल 1 और 2

(d)

1, 2 और 3

नलए सशक्त था। 2.

सोंघ के द्वारा नकए गए सोंगनठत प्रर्ासमों के कारि बौद्ध धमय िे उत्तर र्ारत में तेिी से प्रगनत की थी।

उपर्ुयक्त कथिमों में से कौि-सा/से सही है /हैं ? (a)

केिल 1

(b)

केिल 2

(c)

1 और 2 दमिमों

(d)

ि तम 1, ि ही 2

RAUSIAS-TC19E1007

42

Q.96) Consider the following statements about the Indus Valley Civilization: 1.

The practice of pot burials is found at Lothal, sometimes with pairs of skeletons.

2.

There is clear evidence for the practice of Sati.

3.

The Harappan script has been deciphered.

Which of the statements given above is/are correct?

Q.99) Consider the following statements: 1.

Bimbisara belonged to Haryanka dynasty and consolidated his position matrimonial alliances.

2.

He was a contemporary of both Vardhamana Mahavira and Gautama Buddha.

Which of the statements given above is/are correct? (a)

1 only

(a)

1 only

(b)

2 only

(b)

2 only

(c)

Both 1 and 2

(c)

1 and 2 only

(d)

Neither 1 nor 2

(d)

1, 2 and 3

Q.97) Consider the following statements about the Rig Vedic period: 1.

The Rig Vedic Aryans were pastoral people and their main occupation was cattle rearing.

2.

Their wealth was estimated in terms of their cattle.

Which of the statements given above is/are correct? (a)

1 only

(b)

2 only

(c)

Both 1 and 2

(d)

Neither 1 nor 2

Q.98) Consider the following statements about Buddhism: 1.

The Sangha was governed on democratic lines and was empowered to enforce discipline among its members.

2.

Owing to the organised efforts made by the Sangha, Buddhism made rapid progress in North India.

the he by

Q.100) Consider the following statements about the effects of Alexander’s invasion: 1.

The immediate effect of Alexander’s invasion was that it encouraged political unification of north India under the Mauryas.

2.

Alexander’s invasion had also paved the way for direct contact between India and Greece.

3.

The routes opened by him and his naval explorations increased the existing facilities for trade between India and West Asia.

Which of the statements given above is/are correct? (a)

1 only

(b)

2 only

(c)

1 and 2 only

(d)

1, 2 and 3

Which of the statements given above is/are correct? (a)

1 only

(b)

2 only

(c)

Both 1 and 2

(d)

Neither 1 nor 2

RAUSIAS-TC19E1007

43

Test is part of Rau’s IAS Test series for Preliminary Exam 2019

THEMATIC + CURRENT AFFAIRS

Test Code

GENERAL STUDIES (PAPER –I) THEMATIC TEST–VII

TC19E1007

SUBJECT: AIH + Government Policies + Current Affairs Time Allowed: 2 Hours

Maximum Marks: 200 INSTRUCTIONS

1.

IMMEDIATELY AFTER THE COMMENCEMENT OF THE EXAMINATION, YOU SHOULD CHECK THAT THIS TEST BOOKLET DOES NOT HAVE ANY UNPRINTED OR TORN or MISSING PAGES OR ITEMS, ETC. IF SO, GET IT REPLACED BY A COMPLETE TEST BOOKLET.

2.

This Test Booklet contains 100 items (questions). Each item is printed both in Hindi and English. Each item comprises four responses (answers). You will select the response which you want to mark on the Answer Sheet. In case you feel that there is more than one correct response, mark the response which you consider the best. In any case, choose ONLY ONE response for each item.

3.

You have to mark all your responses ONLY on the separate Answer Sheet (OMR sheet) provided. Read the directions in the Answer Sheet.

4.

All items carry equal marks.

5.

Before you proceed to mark in the Answer Sheet the response to various items in the Test booklet, you have to fill in some particulars in the Answer Sheet as per instructions contained therein.

6.

After you have completed filling in all your responses on the Answer Sheet and the examination has concluded, you should hand over to the Invigilator only the Answer Sheet. You are permitted to take away with you the Test Booklet.

7.

Penalty for wrong answers: THERE WILL BE PENALTY FOR WRONG ANSWERS MARKED BY A CANDIDATE IN THE OBJECTIVE TYPE QUESTION PAPERS. (i)

There are four alternatives for the answer to every question. For each question for which a wrong answer has been given by the candidate, one-third of the marks assigned to that question will be deducted as penalty.

(ii)

If a candidate gives more than one answer, it will be treated as a wrong answer even if one of the given answers happens to be correct and there will be same penalty as above to that question.

(iii)

If a question is left blank, i.e., no answer is given by the candidate, there will be no penalty for that question.

This test is part of Rau’s IAS Test series for Pre liminary Exam 2019

Answers & Explanations of Test Code : TC19E1007 AIH + Gov. Policies + Current Affairs THEMATIC + CURRENT AFFAIRS GENERAL STUDIES (PAPER –I)

ANSWERS & EXPLANATION OF AIH + Gov. Policies + Current Affairs (TC19E1007) Q.1)

Ans. (d)

Explanation:

Explanation:

Q.2)

1.

Samaharta

:

The highest officer in charge of assessment and collection.

2.

Sannidhata

:

The chief custodian of the state treasury and storehouse.



In 1441, Shah Rukh of Persia sent Kamal-ud-din Abdur Razzaq as an emissary to Vijayanagar. Abdur Razzaq wrote about his travels in the Matla-usSadain wa Majma-ul-Bahrain, or ‘The Rise of Two Auspicious Constellations and The Confluence of Two Oceans’.

Q.6)

Ans. (a) Explanation:



Lepakshi paintings related to the Vijayanagar Empire. They were secular, although painted on temple walls. There was complete absence of primary colours, particularly blue. They show a decline in the quality of paintings.

Q.7)

Ans. (d)

Ans. (c) Explanation:



Q.3)

Abhijnanasshakuntalam, written by Kalidasa, relates to the love story of king Dushyanta and Shakuntalam.

Explanation:

Ans. (b) Explanation:



Q.4)

One of the reasons for the popularity of Jainism was the fact that Mahavira preached in the local dialect. Mahavira preached in the vernacular dialect named Ardha Magadhi.



In all four of these sites only the name “Ashoka” is mentioned apart from the name “"Devanam Piyadasi".

Q.8)

Ans. (d) Explanation:



Ans. (c) Explanation:





Q.5)

The Satavahanas find mention in the Puranas, the sacred Hindu texts, but their rise to prominence was led by Simuka, who declared his independence from the Mauryan rule. He then embarked on conquests of Malwa and Maharashtra, and annexed parts of western and central India. The capital of Satvahanas Pratishthana or Paithan. Ans. (d)

RAUS IAS TC 19 E 1007

 

was 

Satavahanas started out as feudatories to the Mauryan dynasty, but declared independence with its decline. They are known for their patronage of Hinduism and Buddhism. The official language of the Satavahans was Prakrit. Among the literary sources, King Hala, who was a poet has written a book in Prakrit language titled as “Gatha Saptasati” which is a collection of folk tales and throw light on social milieu and cultural ethos of the time. The Satavahans started the practice of granting tax-free villages to the Brahmans and the Buddhist monks.

1

Q.9)

Ans. (d)



Bhaja Caves-Maharashtra



Udaygiri Caves- Madhya Pradesh

Explanation: •

Social differentiation is indicated by different residential structures, with the number of rooms varying from one to twelve. There was a citadel where the ruling class lived and the lowest tower was where the common people dwelt.

Q.15) Ans. (c) Explanation: •

Bindusara is also mentioned in different sources with the names like Madrasar, Simhasena and Amitrochates.



Greek sources refer to him as Amitrochates, Greek for the Sanskrit Amitraghata (destroyer of foes). The name perhaps reflects his successful campaign in the Deccan.



Chandragupta—Bindusara’s father and founder of the Mauryan empire—had already conquered northern India. Bindusara’s campaign stopped close to what is today Karnataka, probably because the territories of the extreme south, such as those of the Cholas, Pandyas, and Cheras, had good relations with the Mauryas.

Q.10) Ans. (d) Explanation: •

Biological monitoring of the Ganga has shown that two tributaries, Pandu and Varuna, are increasing pollution load of the river as they are “severely polluted” before their confluence point, a Central Pollution Control Board (CPCB) report has pointed out.

Q.11) Ans. (d) Explanation: •

The Sultan Ganj Buddha is a GuptaPala transitional period sculpture, the largest substantially complete copper Buddha figure known from this time. The statue is dated by archaeologists to between 500 and 700 AD.

Q.16) Ans. (c) Explanation: •

In the Chalcolithic sites copper was also used extensively. Chalcolithic sites were majorly located in Rajasthan, Gujarat, Madhya Pradesh and Maharashtra.

Q.12) Ans. (c) Q.17) Ans. (a)

Explanation: • •

Badryana was associated Uttara Mimansa philosophy.

with

Explanation:

the •

Kapila was the founder of the Sankhya philosophy.

Q.13) Ans. (c)

Q.18) Ans. (a) Explanation:

Explanation: •

The Dharma Chakra Pravartana is said to be the first sermon given by the Buddha after he attained enlightenment. It is taught that the Buddha attained enlightenment while sitting under the Bodhi tree by the river Neranjara, in Bodhgaya.

Milind Panho is a Buddhist text written in Sanskrit. In this text, the Indo- Greek King Menander is mentioned.



Kula refers to the families with common ancestors and kinship. Grama means village. Collection of villages constitute a Vis. Jana refers to a region, while Rashtra means nation.

Q.19) Ans. (c) Explanation:

Q.14) Ans. (c) Explanation:

RAUS IAS TC 19 E 1007



The Harappans carried on long-distance trade in lapis lazuli. Lapis objects may

2

have contributed to the social prestige of the ruling class. •

Mesopotamian records from about 2350 BC onwards refer to trade relations with Meluha, which was the ancient name given to the Indus region.

well versed calculations. •

in

various

kinds

of

Fa-hsien states that the Chandalas lived outside the village and dealt in meat and flesh.

Q.24) Ans. (c) Q.20) Ans. (c) Explanation: •

According to the Arthashastra of Kautiliya, a striking social development of the Mauryan period was the employment of slaves in agricultural operations.



Megasthenes states that he did not notice any slaves in India, but there is little doubt that there had been domestic slaves from the Vedic times onwards. It seems that during the Mauryan period, slaves were engaged in agricultural work on a large scale.

Q.21) Ans. (b)

Explanation: •

In Harsha’s empire, law and order was not well maintained. Hsuan Tsang was robbed of his belongings, although he reports that according to the laws of the land, severe punishments were inflicted for crime.



He calls Shudras agriculturists, which is significant. The earlier texts represent them as serving the three higher Varnas.

Q.25) Ans. (d) Explanation: •

Ratnavali, Priyadarshika and Nagananda were written by Harshavardhana. However, Harsha’s authorship of the three dramas is doubted by several medieval scholars.



It is held that they were composed by a person, called Dhavaka in the name of Harsha for some consideration.

Explanation: •

Lingaraja temple is situated in Odisha.

Q.22) Ans. (c) Explanation: •

Mahavira teaches us that life and destiny of mankind is not the result of the whims of some distant God. According to him, each one of us is what we have made ourselves by our actions in this life and in previous lives. Mahavira believed that all the objects on earth, living or non-living have soul and consciousness. He rejected the authority of the Vedas and supremacy of the Brahmins. Rather, Jainism was a reaction against the Vedic rituals.

Q.23) Ans. (c) Explanation: •

Aryabhata displays awareness of both the zero system and the decimal system. He was a mathematician and

RAUS IAS TC 19 E 1007

Q.26) Ans. (a) Explanation: •

The Kalabhra were of Buddhist persuasion as they patronized Buddhist monasteries.



The kalabhra’s revolt was so widespread that it could be quelled only through the joint efforts of the Pandyas, the Pallavas and the Chalukyas of Badami.

Q.27) Ans.

(d)

Explanation: •

These Centres are intended to support women and not children. Women facing physical, sexual, emotional, psychological and economic abuse, irrespective of age, class, caste, education status, marital status, race

3

and culture will be facilitated with support and redressal.

Q.30) Ans. (a)



The OSCs will support all women including girls below 18 years of age affected by violence, irrespective of caste, class, religion, region, sexual orientation or marital status.





The Ministry of Women and Child Development (MWCD) will provide support to the State Governments/UT Administrations for the establishment of OSC.

In the context of establishing quality residential schools for the promotion of education, Eklavya Model Residential Schools (EMRSs) for ST students are set up in States / UTs with provisioning of funds through “Grants under Article 275(1) of the Constitution”.



The establishing of EMRSs is based on demand of the concerned States/UTs with availability of land as an essential attribute. As per the budget 2018-19, every block with more than 50% ST population and at least 20,000 tribal persons, will have an Eklavya Model Residential School by the year 2022.



The literacy rates of the STs has increased from 8.53% in 1961 to 58.96% in 2011. However, significant disparities still exist in enrolment rates and drop-outs across states, districts and blocks.

Explanation:

Q.28) Ans. (c) Explanation: •



On the occasion of International Women’s Day, the Ministry of Micro, Small and Medium Enterprises (MSME) launched the Udyam Sakhi Portal for women entrepreneurs of India. The portal provides-assistance through its platform for entrepreneurship learning tools, incubation facility, training programmes for fund raising, providing mentors, one-on-one investor meet, provide market survey facility and technical assistance.

Q.31) Ans. (b) Explanation: •

It is an Initiative of the Office of the President of India to recognize, respect and reward grassroots innovations.



FINE would provide platform to the innovators for building the linkages with potential stakeholders, whose support can improve their prospects in coming years for the larger social good. The event was held at the Rashtrapati Bhavan.

Q.29) Ans. (b) Explanation: •

Haryana was the first state to adopt this Scheme.



The aim is to have one such volunteer in every village whose primary job will be to keep an eye on situations where women in the village are harassed or their rights and entitlements are denied or their development is prevented (bridge the gap between Police and Civil society) (Community volunteerism).



It is a joint Initiative of Ministry of Women and Child Development and Ministry of Home Affairs.



In order to provide a link between police and community and facilitate women in distress, one Mahila Police Volunteer (MPV) is envisaged per Gram Panchayat across the country.

RAUS IAS TC 19 E 1007

Q.32) Ans. (b) Explanation: •

It is a national Initiative that will be jointly funded and steered by the Ministry of Human Resource Development and Department of Science and Technology (DST).



While any faculty member from an MHRD funded Higher Education Institute (HEI) including Centrally Funded Technical Institution (CFTI) may lead as the Principal Investigator

4

(PI), single or multiple partners from stake holder ministry, institutions, PSUs, strategic agencies and industry are welcome as Co-PI, partner or collaborator. Q.33) Ans. (d)

other nuclear device, which can do the same job. •

Q.35) Ans. (c)

Explanation: •



The Scheme is targeted at poor, deprived rural families and identified occupational category of urban workers' families. AB-NHPS will have a defined benefit cover of Rs. 5 lakh per family (on a family floater basis) per year for secondary and tertiary care hospitalisation. In order to ensure that nobody is left out (especially women, children and the elderly), there will be no cap on the family size and age under the AB-NHPS.



The beneficiaries under the Scheme will be identified by the SECC (Socio Economic and Caste Census) data. The beneficiaries will not be required to pay any charges and premium for the hospitalisation expenses. The benefit also includes preand posthospitalisation expenses.



Services under the Scheme can be availed at all public hospitals and empanelled private health care facilities.

Q.34) Ans.

Explanation: •

The Ministry of New and Renewable Energy has issued National Wind-Solar Hybrid Policy.



The Policy seeks to promote new hybrid Projects as well as hybridization of existing wind/solar Projects.



On technology front the Policy provides for integration of both the energy sources i.e. wind and solar at AC as well as DC level.



The Policy also provides for flexibility in share of wind and solar components in hybrid Project, subject to the condition that, rated power capacity of one resource be at least 25 per cent of the rated power capacity of other resource for it to be recognised as a hybrid Project.

Q.36) Ans. (c) Explanation: •

Recently, the Cabinet Committee on Economic Affairs approved continuation of Umbrella Scheme ‘Green Revolution — Krishonnati Yojana’ in Agriculture Sector.



The Schemes that are part of the Umbrella Schemes are:

(b)

Explanation: •





It is NASA’s mission that plans to shunt or blow up threatening space rocks. Earth is hit by asteroids with surprising regularity but most of them are too small to do much damage or they fall in unpopulated areas. HAMMER (Hypervelocity Asteroid Mitigation Mission for Emergency Response vehicle), the deflector is nine metres tall and weighs more than 8000 kg. HAMMER can be used as a kinetic impactor (spacecraft at a high speed to give the push), or as a carrier for some

RAUS IAS TC 19 E 1007

In a new study, NASA used the asteroid Bennu as a hypothetical test for the HAMMER programme.

-

Mission Development (MIDH)

for of

Integrated Horticulture

-

National Food Security Mission (NFSM), including National Mission on Oil Seeds and Oil Palm (NMOOP)

-

National Mission for Sustainable Agriculture (NMSA)

-

Sub-Mission on Extension (SMAE)

Agriculture

5



-

Sub-Mission on Seeds Planting Material (SMSP)

-

National (NeGP-A)

-

Integrated Scheme on Agricultural Marketing (ISAM)

e-Governance

and

as chosen by the subscriber at the time of purchase. The deposits made in the Scheme are exempted from income tax under section 80C of the Income Tax Act, 1961.

Plan

The Mission for Integrated Development of Horticulture (MIDH) aims to promote holistic growth of horticulture sector; to enhance horticulture production, improve nutritional security and income support to support farm households.

Q.39) Ans. (a) Explanation: •

Q.37) Ans. (a) Explanation: •



Green skills contribute to preserving or restoring environmental quality for sustainable future and include jobs that protect ecosystems and biodiversity, reduce energy and minimize waste and pollution. It has been developed by the Ministry of Environment, Forest and Climate Change in consultation with the National Skill Development Agency (NSDA). NSDA is the nodal agency for synergizing skill development Initiatives in the country, under the Ministry of Skill Development and Entrepreneurship (MSDE).

Q.40) Ans. (b) Explanation: •

The maternity benefits under Pradhan Mantri Matru Vandana Yojana (PMMVY) are available to all Pregnant Women and Lactating Mothers (PW and LM) except those in regular employment with the Central Government or State Government or Public Sector Undertaking or those who are in receipt of similar benefits under any law for the time being in force.



This Scheme is applicable only to the first living child of the family.



Eligible beneficiaries will get Rs. 5,000 under PMMVY and the remaining cash incentive as per approved norms towards Maternity Benefits under the Janani Suraksha Yojana (JSY).

Q.38) Ans. (c) Explanation: •



The Scheme provides social security during old age and protects elderly persons aged 60 and above against the future fall in their interest income due to uncertain market conditions. The Scheme provides an assured return of 8% per annum for 10 years. The differential return, i.e. the difference between return generated by LIC and the assured return of 8% per annum would be borne by the Government of India as subsidy on an annual basis. Pension is payable at the end of each period during the policy tenure of 10 years, as per the frequency of monthly/quarterly/ half-yearly/yearly

RAUS IAS TC 19 E 1007

Under this Mission, the Ministry of Micro Small and Medium Enterprises (MSME) will cover 50 clusters and each cluster will employ 400 to 2,000 artisans. It also aims at linking five crore women across the country to the Initiative.

Q.41) Ans. (c) Explanation: •

Launched by the Ministry of Water Resources the Scheme is aimed at sustainable ground water management with community participation in select over-exploited and ground water stressed areas in Seven States (Gujarat, Haryana, Karnataka, Madhya Pradesh, Maharashtra, Rajasthan and Uttar Pradesh).



ABHY is designed as a Central Sector Scheme and is proposed to be

6

implemented assistance.

with

World

Bank’s

Q.42) Ans. (d) Explanation: •

A National Consultation on the National Rural Drinking Water Programme (NRDWP) and Swajal was held in the capital today to discuss the reforms needed in NRDWP and to outline a road map for the Swajal Scheme.

Q.43) Ans.(b)



Indian scientist-academician, N. Raghuram, had been elected to the Chair of the International Nitrogen Initiative (INI).



The programme is currently a sustained partner of Future Earth.

Q.45) Ans. (b) Explanation: •

The Scheme is named as NextGen Airports for Bharat (NABH)



This Scheme constitutes investments to be made in airport upgrade by both the private sector and the state-owned Airports Authority of India (AAI) in due course of time.

Explanation: •





The system has been developed and hosted by the Ministry of Environment, Forest and Climate Change, with technical support from National Informatics Centre, (NIC). The main highlights of PARIVESH include - single registration and single sign-in for all types of clearances (i.e. Environment, Forest, Wildlife and CRZ), unique-ID for all types of clearances required for a particular Project and a single Window interface for the proponent to submit applications for getting all types of clearances. It has a Single Window System for Central, State and District level clearances.

Q.46) Ans. (b) Explanation: •

The Scheme is aimed at ensuring remunerative prices to the farmers for their produce as announced in the Union Budget for 2018. It will provide MSP assurance to farmers-A reflection of Government’s commitment to the “Annadata”



The new Umbrella Scheme includes the mechanism of ensuring remunerative prices to the farmers and is comprised of

Q.44) Ans. (d) Explanation: •

The International Nitrogen Initiative (INI) is an international programme, set up in 2003 under sponsorship of the Scientific Committee on Problems of the Environment (SCOPE) and from the International Geosphere-Biosphere Programme (IGBP). The key aims of the INI are to: -

-

optimize nitrogen’s beneficial role in sustainable food production, and minimize nitrogen’s negative effects on human health and the environment resulting from food and energy production.

RAUS IAS TC 19 E 1007



-

Price Support Scheme (PSS),

-

Price Deficiency Payment Scheme (PDPS)

-

Pilot of Private Procurement and Stockist Scheme (PPPS).

It is also decided that in addition to NAFED, Food Corporation of India (FCI) will take up PSS operations in states /districts. The procurement expenditure and losses due to procurement will be borne by Central Government as per norms.

Q.47) Ans: (a) Explanation: •

The Project was launched with World Bank Assistance.

7



In addition to rehabilitation of dams, other important activities include Design Flood Review, publication of important Guidelines as well as Manuals dealing with Dam Safety Management, preparation of O and M Manuals, Emergency Action Plans, development of web based asset management tool i.e. Dam Health And Rehabilitation Monitoring Application (DHARMA), Seismic Hazard Mapping along with development of Seismic Hazard Assessment Information System (SHAISYS), Risk Assessment of few selected dams, organisation of Dam Safety Conferences, national and international training programmes, technical exposure visits etc.

requirements provided.



Google is using its expertise in artificial intelligence for Project Navlekha, using which, it will quickly render any PDF with Indian language content into editable text, overcoming issues that usually occur when you try to copy text in Indian languages from a PDF.

Q.49) Ans. (c) The Scheme envisages to provide last mile connectivity and electricity connections to all remaining households in rural as well as urban areas.

are



Ashoka appointed Rajukas for the administration of justice in his empire.



The Yuktas, the Rajukas and the Pradesikas who were used to be sent on tour around the kingdom every five years seem to have acted as healthy checks.

Q.52) Ans. (c) Explanation: •

Megaliths are spread across the Indian sub-continent, though the bulk of them are found in peninsular India, concentrated in the states of Maharashtra (mainly in Vidarbha), Karnataka, Tamil Nadu, Kerala, Andhra Pradesh and Telangana.



Megalithic culture finds several references in ancient Tamil Sangam literature. For instance, Menhirs are referred to as Nadukal (memorial stone).



Megalithic people used various types of pottery including red ware. But black and red ware seems to have been popular with them.

Q.53) Ans. (a) Explanation:

Explanation: •

airport

Explanation:

Explanation: The Project Navlekha was launched by Google. Navlekha, in Sanskrit means "a new way to write".

the

Q.51) Ans. (a)

Q.48) Ans. (a) •

at



Hemis Monastery is a Himalayan Buddhist monastery of the Drukpa Lineage, in Hemis, Ladakh, India.

Q.54) Ans. (c) Explanation:

Q.50) Ans. (c) Explanation: •

The Initiative mandates optional linking of Aadhaar to airlines at the time of booking for faster airport entry and automated check-ins without requiring any paper-based interventions.



A biometrics-based digital processing system for passenger entry and related

RAUS IAS TC 19 E 1007



Muziris Heritage Project’s objectives: -

To promote understanding distinctiveness Muziris

awareness and of the cultural and diversity of

-

To conserve cultural assets and safeguard them for the present and future generations

8

-

To practice and sustainable development

promote

-

To promote participative approach towards conservation and restoration

-

To ensure accessibility for all

-

To include the heritage of Muziris in regional educational programmes

-

To integrate and heritage management for mutual synergy

respective States/Union Territories to wind up a co-operative bank or to supersede its committee of management and requiring the Registrar not to take any action for winding up, amalgamation or reconstruction of a cooperative bank without prior sanction in writing from the RBI, are treated as eligible banks. •

Q.55) Ans. (a) Explanation: •





Recently, two antique statues worth hundreds of thousands of dollars stolen from India and displayed at two American museums have been repatriated to India by the US. The first statue, ‘Lingodhbhavamurti’, a 12th century granite sculpture depicting an iconic representation of Lord Shiva, dates back to the Chola dynasty. The second phyllite sculpture depicts the bodhisattva of wisdom, ‘Manjusri’, holding a sword and painted in gold leaf.

Q.57) Ans. (c) Explanation: •

Himalayan Griffon Vulture is listed as ‘Near Threatened’ in the IUCN Red List. Other species of vultures such as the long-billed, slender-billed and whiterumped are listed as “Critically Endangered” in the IUCN Red List.



It is protected in Schedule IV of the Wildlife Protection Act 1972.



Additional Information – Vultures protected in the Schedule I of the Wildlife Protection Act 1972 are:

Q.56) Ans. (a) Explanation: •



Banks covered by Deposit Insurance and Credit Guarantee Corporation (i)

All commercial banks including the branches of foreign banks functioning in India, Local Area Banks and Regional Rural Banks.

(ii)

Co-operative Banks - All eligible co-operative banks as defined in Section 2(gg) of the DICGC Act are covered by the Deposit Insurance Scheme.

All State, Central and Primary cooperative banks functioning in the States/Union Territories which have amended their Co-operative Societies Act as required under the DICGC Act, 1961, empowering RBI to order the Registrar of Co-operative Societies of the

RAUS IAS TC 19 E 1007

At present all Co-operative banks are covered by the Scheme. The Union Territories of Lakshadweep and Dadra and Nagar Haveli do not have Cooperative Banks.



-

Long billed Vulture - Gyps indicus.

-

Slender-Billed Vulture -Gyps Tenuirostris.

-

White-Rumped Vulture -Gyps Bengalensis.

Recently, two Himalayan griffon vultures, which were poisoned at Patiagaon near the Brahmaputra bank, were released at Kaziranga National Park after being treated at the Centre for Wildlife Rehabilitation and Conservation.

Q.58) Ans. (a) Explanation: •

The painted stork is a large wader in the stork family. It is found in the wetlands of the plains of tropical Asia south of the Himalayas in the Indian Subcontinent and extending into Southeast Asia.

9



Its scientific leucocephala.

name

is

Mycteria





It is listed in Schedule IV of the Wildlife Protection Act 1972.

As a biodiversity discovery tool, DNA barcoding helps to flag species that are potentially new to science.



As a biological tool, DNA barcoding is being used to address fundamental ecological and evolutionary questions, such as how species in plant communities are assembled.



The process of DNA barcoding entails two basic steps: (1) building the DNA barcode library of known species; and (2) matching the barcode sequence of the unknown sample against the barcode library for identification.

Q.59) Ans. (d) Explanation: •

The 2018 Environmental Performance Index (EPI) finds that air quality is the leading environmental threat to public health. Now in its twentieth year, the biennial report is produced by researchers at the Yale and Columbia Universities in collaboration with the World Economic Forum.



The tenth EPI report ranks 180 countries on 24 performance indicators across 10 issue categories covering environmental health and ecosystem vitality. Switzerland leads the world in sustainability.

Q.61) Ans. (b) •

The large Nalabana Island (Forest of Reeds) covering about 16 sq km in the lagoon area was declared a Bird Sanctuary in 1987.

Switzerland’s top ranking reflects strong performance across most issues, especially air quality and climate protection.



The lake was included in the Montreax Record (Threatened) list in 1993.



However, it was removed from the Montreax record in 2002 due to successful restoration by Chilika development authority. It was the first site to be removed from Montreaux record.



It is home to the Irrawady Dolphin, which is listed as endangered under IUCN.







In general, high scorers exhibit longstanding commitments to protecting public health, preserving natural resources and decoupling greenhouse gas (GHG) emissions from economic activities. India and Bangladesh come at near the bottom of the rankings, with Burundi, Democratic Republic of the Congo, and Nepal rounding out the bottom five.

Q.60) Ans. (c)

Explanation:

Q.62) Ans. (c) Explanation: •

The Supreme Court in a recent judgment added procedural safeguards for accused under the Scheduled Castes and Scheduled Tribes (Prevention of Atrocities) Act, 1989.



The Supreme Court gave the following orders:

Explanation: •



DNA barcoding, a new method for the quick identification of any species based on extracting a DNA sequence from a tiny tissue sample of any organism, is now being applied to taxa across the tree of life. As a research tool for taxonomists, DNA barcoding assists in identification by expanding the ability to diagnose species by including all life history stages of an organism.

RAUS IAS TC 19 E 1007

-

-

It held that government officials cannot be prosecuted on mere allegation of committing an offence under the Act without the sanction of appointing authority. The Court said that a preliminary enquiry must be conducted before

10

-

-

registering an FIR (First Information Report) to check: Whether the case falls within the parameters of the Atrocities Act and Whether the case is frivolous or motivated.



The court directed that public servants can only be arrested with the written permission of their appointing authority.



In the case of other employees, no arrest can be made without the approval of the District Senior Superintendent of Police.

Q.63) Ans. (b)

Regime (MTCR), Wassenaar Arrangement (WA), Australia Group (AG) and the Nuclear Supplier Group (NSG). Q.65) Ans. (c) Explanation: •

A biomarker or biological marker is in general a substance used as an indicator of a biological state. It is a characteristic that is objectively measured and evaluated as an indicator of normal biological processes, pathogenic processes, or pharmacologic response to a therapeutic intervention. It is used in many scientific fields.



Ideally, an ideal marker should be safe and easy to measure, cost efficient to follow up, modifiable with treatment, consistent across genders and ethnic groups.



It can be introduced into an organism as a means to examine organ function or other aspects of health. For example, rubidium chloride is used in isotopic labeling to evaluate perfusion of heart muscle. It can also be a substance whose detection indicates a particular disease state, for example, the presence of an antibody may indicate an infection.

Explanation: •



According to the International Monetary Fund, “Remittances represent household income from foreign economies arising mainly from the temporary or permanent movement of people to those economies.” India is the largest remittances receiving country in the world. It accounts for the second largest item in the country’s Balance of Invisibles (BoI), which is a part of the Current Account.

Q.64) Ans. (a) Explanation: •

The US Government has decided to move India into Tier-1 of its Strategic Trade Authorization license exception, third in Asia after Japan and South Korea, and first in South Asia.



This authorises the export, re-export and in-country transfer of specific sensitive defence technologies and dual use technologies to destinations that the US regards as low risk.



India is the 37th country to be designated the STA-1 status by the United States.



Traditionally, the US has placed only those countries in the STA-1 list who are members of the four export control regimes: Missile Technology Control

RAUS IAS TC 19 E 1007

Q.66) Ans. (b) Explanation: •

Geckos are reptiles and are found on all the continents except Antarctica.



It is important to note that Gecko tails serve many purposes. They help balance their weight as they climb branches, they act as fuel tanks to store fat, and as camouflage to help them disappear into their environment.



Geckos are also able to shed their tails if a predator grabs them.



Most geckos are nocturnal, which means they are active at night, but Day Geckos are active during the day and nibble on insects, fruits, and flower nectar. Most Geckos make noises, such

11

as chirping, barking and clicking when they are defending their territory or attracting a mate. •

-

-

Hemidactylus Triedrus (The southern termite hill gecko)Nellore, Puducherry and Sri Lanka.



Hemidactylus Whitakeri (in honour of herpetologist Romulus Whitaker’s contributions to the conservation of India’s reptiles)South Karnataka and the Nilgiris district in Tamil Nadu. Hemidactylus Sahgali -Geckos from Maharashtra, Gujarat and parts of Pakistan.

Q.67) Ans. (a) Explanation: •

Sustainable development of islands and overall maritime development havebeen accorded high priority by the government.



The Island Development Agency (IDA), an apex body, under the chairmanship of the Home Minister, was constituted in June 2017.



Convergence (of Central and State Schemes);

(ii)

Collaboration (among citizens and functionaries of the central and state governments, including districts); and

(iii)

Competition (among districts)

The three categories: -



(i)

NITI Aayog has been mandated to steer the Holistic Development of Islands programme, along with the respective UT/state governments. Under the said programme in the first phase, 10 Islands in the Andaman & Nicobar and Lakshadweep have been taken up for holistic development.

Niti Aayog is the nodal agency for the Aspirational District Programme.



A total of 115 districts have been selected for the programme.



The core principles of the programme are:

RAUS IAS TC 19 E 1007

for

the

-

Health and nutrition;

-

Education;

-

Agriculture and water resources;

-

Financial inclusion;

-

Skill development; and

-

Basic infrastructure

For each district, a Prabhari Officer of the rank of Additional Secretary/Joint Secretary has been nominated.



Delta Ranking is done for tracking and monitoring various developmental gains for these districts. Third party monitoring of the data will be done by Tata Trusts and IDinsight, in 76 and 25 districts, respectively.

Q.69) Ans. (b) Explanation: •

The Van Dhan Yojana will be implemented through the Ministry of Tribal Affairs as nodal department at the central level and TRIFED as nodal agency at the national level.



At the unit level, aggregation of minor forest produce would be done by SHGs having about 30 members each, forming Van Dhan Vikas ‘Samuh’. The SHGs would also undertake primary value addition of the MFPs using equipment, such as small cutting and sieving tools, decorticator, dryer, packaging tool, etc. based on MFPs available in the area.



Under Van Dhan, 10 Self Help Groups of 30 tribal gatherers is constituted. The establishment of “Van Dhan Vikas Kendra” is for providing skill upgradation and capacity building training, and setting up of primary processing and value addition facility. They are then trained and provided with

Q.68) Ans. (c) •

areas



NITI Aayog hosted the Investers’ Conference for Holistic Development of Islands in August, 2018.

Explanation:

The main focus programme are:

12

working capital to add value to the products, which they collect from the jungle. •

Telecommunications Union (ITU) in its ‘IMT 2020’ specification. •

The 3GPP (3rd Generation Partnership Project), an industry driven standardization body, that has undertaken the standardization of mobile technologies for the past 25 years, is currently developing standards for 5G networks based on the ITU requirements.



3GPP has proposed three main technology categories and corresponding use cases for the 5G.

Benefits of the Scheme: -

Value addition of non-timber forest produce (NTFP) would lead to two to three times increase in tribal incomes from NTFPs.

-

Eliminate multiple middlemen.

-

Increase shelf life of the produce and reduce wastage.

layers

of

Q.70) Ans. (a)

(i)

eMBB – Enhanced Mobile Broadband: Supports ultra-high level speed connections indoors and outdoors.

(ii)

mMTC – Massive Machine Type Communications: Supports a very large number of connected devices, usually called Internet of Things with varying quality of service requirements.

(iii)

uRLLC – Ultra-reliable and Low Latency Communications: Latency of less than one milli-second and low pocket loss rates of better than one in 10,000 packets.

Explanation: •

The MSP is announced by the Ministry of Tribal Affairs, based on the recommendations of the Pricing Cell constituted by TRIFED (Tribal Cooperative Marketing Development Federation of India).



Currently, MSP is announced for 49 minor forest produce.



MSP declared by the Government of India (GOI) shall be reference MSP for fixing MSP and state governments shall have latitude of 10% of MSP declared by GOI, i.e., the States can fix MSP upto 10% higher or lower than MSP declared by GOI.



The state is also at liberty to suspend process of procurement or not to do procurement of MFP at all.



The area of coverage of the Scheme has been extended beyond existing 9 Schedule 5 states and now Scheme is applicable in all states.

Q.71) Ans. (d) Explanation: •

The Department of Telecommunications (DOT) had constituted a high level forum for 5G technology in India. It is headed by the Secretary, DOT.

• The steering committee of the high level forum is headed by A.J. Paulraj. •

The macro level requirements for 5G have been defined by the International

RAUS IAS TC 19 E 1007

Q.72) Ans. (a) Explanation: •

The Deen Dayal Sparsh Yojana is a panIndia scholarship programme for school children to increase the reach of philately. Under the Scheme, it is proposed to award annual scholarships to children of standard VI to IX having good academic record and also pursuing Philately as a hobby through a competitive selection process in all postal circles. Briefing the media after the launch of the Scheme, the Minister said that under the Scheme, it is proposed to award 920 scholarships to students pursuing philately as a hobby. Every postal circle will select a maximum of 40 scholarships representing 10 students each from Standard VI, VII, VIII and IX. The amount of scholarship will be Rs.

13

6,000/- per annum @ Rs. 500/- per month. Q.73) Ans. (d)



To provide demand driven, placement oriented National Skills Qualifications Framework (NSQF) compliant skilling programmes to incentivize and supplement the efforts of the industry in creating jobs in the organized textile and related sectors, covering the entire value chain of textiles, excluding spinning and weaving.

Providing well-fitting garments in the absence of standardized size chart is proving to be a big challenge for the domestic textile and apparel industry, which is projected to reach USD 123 billion by 2021 and holds 5th position in apparel imports.



To promote skilling and skill upgradation in the traditional sectors of handlooms, handicrafts, sericulture and jute.

The findings of the study will impact various other sectors, like automotive, aerospace, fitness and sport, art and computer gaming, where insights from this data can produce ergonomically designed products which are suited for the Indian population.



NIFT is the implementing agency for the National Sizing Survey.

Explanation: •

Objectives of the SAMARTH Scheme are: (i)

(ii)

(iii)



returns of the garments are in the range of 20% to 40% and is increasing with the growth of e-commerce and the main reason for returns are poor garment fit.

To enable provision of sustainable livelihood either by wage or selfemployment, to all sections of the society across the country.

Q.75) Ans. (c) Explanation: •

The Directorate General of Mines Safety (DGMS) is a regulatory agency under the Ministry of labour and employment, Government of India, in matters pertaining to occupational safety, health and welfare of persons employed in mines (coal, metalliferous and oilmines).



Under the Constitution of India, safety, welfare and health of the workers employed in the mines are the concern of the Central Government (Entry 55Union List-Article 246). The objective is regulated by the Mines Act, 1952 and the Rules and Regulations framed there under. These are administered by the Directorate-General of Mines Safety (DGMS), under the Union Ministry of Labour and Employment. Apart from administering the Mines Act and the subordinate legislation there under, DGMS also administers a few other allied legislations, including the Indian Electricity Act.



It is headquartered Jharkhand.

The Scheme would target to train 10 lakh persons (9 lakh in organised and 1 lakh in traditional sector), over a period of 3 years (2017-20).

Q.74) Ans. (a) Explanation: •

INDIA SIZE Project was sanctioned by the Ministry of Textiles, the Government of India, under the Research and Development Scheme to develop Indian size charts based on body measurements of Indian population for better fitting of ready to wear garments. The Project ‘India Size’ plans to measure 25,000 (Twenty Five Thousand) persons aged from 18 to 65 years in six different regions of India using 3D body scanners. It is expected that the retail garment industry will be hugely benefitted from the outcome of the Project.



The Indian apparel industry uses size charts, which are tweaked versions of size charts of other countries, so

RAUS IAS TC 19 E 1007

at

Dhanbad,

14

Q.76) Ans. (d)

Scheme of ESIC ‘Atal Bimit Vyakti Kalyan Yojana’. The ESI Corporation (Employees’ State Insurance Corporation) has approved the ‘ATAL BIMIT VYAKTI KALYAN YOJNA’ for Insured Persons (IPs) covered under the Employees’ State Insurance Act, 1948. This Scheme is a relief payable in cash directly to the bank account in case of unemployment and while they search for new engagement. The cash benefit given to the unemployed persons searching for new employment will be 25 percent of his average earning of 90 days.

Explanation: •

A Fixed Term Employee is defined as a workman who is employed on a contract basis for a fixed period. Thus, the services of the workman will be automatically terminated as a result of non-renewal of the contract between the employer and the workman concerned. Separation of service of a workman, as a result of non-renewal of the contract of employment between the employer and workman concerned, shall not be construed as termination of employment.



Impact of inclusion of the term: On the termination of the Fixed Term Employment of the workman, the workman is not entitled to any notice or pay in lieu thereof. However, by proposed inclusion of the Fixed Term Workman as one of the categories of workman in the classification of workman in the Industrial Employment (Standing Orders) Act, 1946, the Ministry of Labour and Employment intends to make such workman on Fixed Term Employment, eligible for all statutory benefits available to a permanent workman, proportionately accordingly to the period of service rendered by him, even though, his period of employment does not extend to the qualifying period of employment required in the statute.





Initially, the concept of Fixed Term Employment was limited to apparel sector. However, later it was extended to all sectors for improving ‘Ease of Doing Business’ for firms intending to hire people for completing specified Projects, tasks and orders. However, firms have been barred from converting permanent posts already existing as Fixed Term Employment.

Q.77) Ans. (d) Explanation: •



ESIC functions under the Ministry of Labour and Employment.



Provident Fund is EPFO and not ESIC.

administered

by

Q.78) Ans. (c) Explanation: •

These ports are being developed under the Sagarmala Project. (i)

Vadhavan (Maharashtra)

(ii)

Enayam (Tamil Nadu)

(iii)

Tajpur (West Bengal)

(iv)

Paradip Outer Harbour (Odisha)

(v)

Sirkazhi (Tamil Nadu)

(vi)

Belekeri (Karnataka)

Q.79) Ans. (d) Explanation: •

The PCPIR Policy is being implemented by the Ministry of Chemicals and Fertilizers.



Four PCPIRs have been identified: -

Gujarat - Dahej

-

Andhra Pradesh – VishakhapatnaKakinada region

-

Odisha - Paradip

-

Tamil Nadu – Nagapattinam

Cuddalore

and

Q.80) Ans. (c) Explanation:

Around 3.2 crore Insured Persons (IPs) will benefit from the newly launched

RAUS IAS TC 19 E 1007

15



GST Network is the technology backbone of the GST indirect tax reform. Initially, it was created as a majority privately held company, with 51% private ownership and 49% public ownership. However, considering the strategic nature of the GST Network, the Cabinet has decided to transform it into a 100% public company, with equal ownership of both the centre and the states.

-

Ordinarily lived, resided or cohabited with a person who was HIV positive.

-

The Act provides for no discrimination against the protected person on any ground such as: 

Q.81) Ans. (b)



Explanation: •

The Cabinet has approved the restructuring of the National Health Agency as National Health Authority for better implementation of the PM-Jan Aarogya Yojana.



The National Health Authority will be an attached office to the Ministry of Health and Family Welfare.



The Governing Board of the National Health Authority will be chaired by the Minister of Health and Family Welfare, Government of India.





The composition of the Governing Board is broad based with due representations from the Government, domain experts, etc.

   •

The Act defines protected persons as a person who is: -

HIV positive or

-

Ordinarily living, residing or cohabiting with a person who is an HIV positive person

RAUS IAS TC 19 E 1007

Upon a competent courts order

-

In case informed consent has been given by the person

The Act provides for Right of Residence to every protected person.



Every state government shall appoint one or more Ombudsman who will receive complaint, inquire into the violations of the provisions of the HIV and AIDS (Prevention and Control) Act, in relation to acts of discrimination and providing of healthcare services.

Explanation:



-



Besides, the states shall also be represented in the Governing Board on a rotational basis.

HIV and AIDS (Prevention and Control) Act, 2017 was enacted for the prevention and control of the spread of HIV and AIDS, and for the protection of human rights of persons affected by the said virus and syndrome.

Disclosure of HIV status: No person shall be compelled to disclose his/her HIV status. However, on following grounds his/her HIV status can be revealed:

Q.82) Ans. (b) •

Denial of, or termination from, employment, unless in the case/of termination, the person, who is otherwise qualified, is furnished with Denial in the opportunity to stand for or hold public or private office No denial of insurance No isolation and segregation HIV testing as a pre-requisite etc.

Q.83) Ans. (b) Explanation: •

The Partnership for Maternal, Newborn and Child Health (The Partnership, PMNCH) is an alliance of more than 1000 organisations in 192 countries from the sexual, reproductive, maternal, newborn, child and adolescent health communities, as well as health influencing sectors. The Partnership provides a platform for organisations to align objectives, strategies and

16

resources, and agree on interventions to improve maternal, newborn, child and adolescent health. •

The Partnership was formed in 2005. Today, the Partnership represents ten constituencies: partner countries; donors and foundations; intergovernmental organisations; nongovernmental organisations; academic, research and training institutions; adolescents and youth; healthcare professional associations; private sector partners, UN agencies and global financing mechanisms.



The Secretariat of the Partnership for Maternal, Newborn and Child Health is hosted and administered by the World Health Organisation in Geneva and undertakes the day-to-day administration of The Partnership’s workplan.



The PMNCH Partners’ Forum serves as a regular global platform for the renewal of commitment to the mission and purpose of The Partnership, for global high level advocacy and for achieving broad consensus on the strategy and priorities of The Partnership.







The Index will grade States and UTs, thus allowing for more than one State/UT to occupy the same grade UTs and therefore all 36 States and UTS to ultimately reach the highest level. The PGI has been conceptualised as a tool to encourage States and UTS to adopt certain practices, like online recruitment and transfer of teachers, electronic attendance of students and teachers etc.



The PGI has seventy (70) indicators divided into two categories, viz., ‘Outcomes’ and ‘Governance’ processes. The first category is divided into four domains, viz., Learning outcomes; Access outcomes; Infrastructure and Facilities and Equity outcomes; the second category is about Governance processes which covers attendance, teacher adequacy, administrative adequacy, training, accountability and transparency.



The total weightage under PGI is 1000 points. Each indicator has been given either ten or twenty points.



The data for the year 2017-18 has been received and compiled for all the State/UTs and final grading has been done after third party verification of data by UNICEF.

Till date, 4 Partners’ Forum meetings have taken place: (i)

2007 Partners’ Forum – Dar es Salam, Tanzania

(ii)

2010 Partners’ Forum – New Delhi, India

Q.85) Ans. (b)

(iii)

2014 Partners’ Forum Johannesburg, South Africa





(iv)

2018 Partners’ Forum – New Delhi, India.

The PM Adarsh Gram Yojana is a Scheme launched by the Ministry of Social Justice and Empowerment.



The Scheme focuses on the integrated development of SC pockets.



According to the census of 2011, the population of SCs is 20.14 cr. which, as mentioned above, constitutes 16.6% of the total population. The highest proportion of SC population is in Punjab.



According to Census 2011, 46,844 villages have >50% of SC population.



To enable an area approach, the PM Adarsh Gram Yojana was launched in

The Partners’ Forum meeting in 2018, in New Delhi was the 4th meeting and first time when it was hosted by a country twice.

Explanation:

Q.84) Ans. (b) Explanation: •

The Performance Grading Index (PGI) has been launched by the Ministry of Human Resource and Development.

RAUS IAS TC 19 E 1007

17

2009-10 on a pilot basis. The Scheme aims at integrated development of villages in which the population of SCs is above 50% and the total population is more than 500. •



The objectives of the Scheme are: -

Provisions of adequate infrastructure necessary for socioeconomic development are provided.

-

Improvement of socio-economic indicators, known as monitorable indicators, are to be improved, so that the disparity between SC and non-SC population is eliminated, and the level of indicators is raised to at least that of the national average.

-

Implementation of the existing Schemes of the Central and the State governments in a convergent manner

Explanation:



In partnership with the state government, the Department of Rural Development has completed ranking of 50,000 Gram Panchayats on parameters of physical infrastructure, human development and economic activities.



The Scheme is launched by the Ministry of Rural Development.

Q.87) Ans. (d) Explanation: •

Government e-Marketplace is a national public procurement portal. It aims to create a public procurement system that is transparent, efficient and inclusive. It is an end-to-end online marketplace for central and state governments, CPSUs, autonomous institutions and local bodies.



It was created after transforming the Directorate General of Supplies and Disposals under the Ministry of Commerce and Industry to a digital ecommerce portal for procurement and selling of goods and services.

Gap Filling Fund of Rs. 20 lakh to the village, where specific interventions can be made, which are currently not addressed by any of the existing Schemes.

Q.86) Ans. (d) Mission Antyodaya is a convergence framework for measurable effective outcomes on parameters that transform lives and livelihoods. It seeks to converge government interventions with Gram Panchayats as the basic unit for planning by following a saturation approach by pooling resources – human and financial – to ensure sustainable livelihoods.

Q.88) Ans. (b) Explanation: •

NPPA was established in 1997 through a resolution of the Ministry of Chemicals and Fertilizers. It exercises its powers under the Essential Commodities Act.



It oversees the implementation of the Drug Price Control Order (DPCO).



It fixes the prices of scheduled drugs, the formula of which is given by DPCO.

The objectives of the Scheme are: -

-

Carry out the baseline survey of households and monitor the progress periodically; Ensure convergence of programmes towards development of rural areas;

RAUS IAS TC 19 E 1007

Institutionalise partnerships at Gram Panchayats, community organisations, NGOs, SHGs, institutions and field-level functionaries of different departments, such as ANMs, ASHA, Anganwadi workers and Promote enterprise through partnerships with institutions and professionals.



Approach and strategy of the Scheme: -



-

18



It also monitors the prices of decontrolled drugs and formulations.

Q.89) Ans. (d)

of Guru Padmasambhava, the founder of Tibetan Buddhism, at Jirang in Gajapati district(Odisha). •

This stone statue is placed in the middle of ‘Padma Sarovar’, a large tank near Padmasambhava Mahavihara, the largest Buddhist monastery in eastern India. The Dalai Lama had inaugurated this monastery.



Historians claim that Guru Padmasambhava, also known as the Second Buddha, was born and brought up in Odisha before he left for Tibet.



The statue of Guru Padmasambhava has been sculpted by the noted sculptor Padmashri Prabhakar Moharana.

Explanation: •

Mattanchery Palace was built around 1555 A.D. by the Portuguese.



It is a two tiered quadrangular building consisting of long and spacious halls. The central open courtyard enshrines the tutelary diety of the royal family “Pazhayannur Bhagwati”. The upper story consists of a coronation hall, bed chamber, known as Ramayana room, dining hall, assembly hall and the staircase.



This palace is located in Kerala.

Q.90) Ans. (c) Explanation: •

Bishnupur temples and built in bricks and also in laterite. The temples at Bishnupur are mostly of Eka-Ratna type with a single tower upon a sloping roof and a square cell (Garbhagriha) flanked by a porch on each side with multicusped arches (i.e., Lalji, Kalachand, Radhashyam, Jormandir and Nandalal).



Bengal style of architectures may be classified into two groups: ‘Chala’ type and ‘Ratna’ type.



Bishnupur temples belong to the second category, which have towers or ‘Sikhara’ resembling spires with faceted sides on the top of the roof.



Depending on the number of ‘Sikharas’, the temples are called ‘Eka-Ratna’, ‘Pancha-Ratna’ or ‘Nava-Ratna’. Temple walls are usually decorated with terracotta panels of floral and geometric patterns, and also with figures depicting contemporary life and, more commonly, mythological stories from the ‘Puranas’.

Q.92) Ans. (d) Explanation: •

In ‘6+1 Group’, Afghanistan, China, India, Iran, Pakistan, Russia and the US are included.



It assumed that each of these countries was essential for the achievement of a political settlement in Afghanistan.

Q.93) Ans. (d) Explanation: •

The Garo Hills Conservation Area (GHCA) comprises three legally designated Protected Areas - Nokrek National Park, Balpakram National Park and Siju Wildlife Sanctuary and the Reserved Forests of Tura Peak, lmangiri, Rewak and Baghmara.



The GHCA lies in Biogeographic Zone 9B (North-eastern India) and is also the western most limit of the Indo-Malayan Biodiversity Hotspots.



Garo Hills are bounded on the north by the Goalpara district of Assam, on the south it shares international boundary with Bangladesh, on the east by the district of Khasi Hills, Meghalaya and Kamrup, Assam and on the west by the district of Goalpara, Assam and Bangladesh.



It is listed in the tentative list of the World Heritage Lists.

Q.91) Ans. (c) Explanation: •

Recently, Odisha Chief Minister Naveen Patnaik unveiled a 19-foot-high statue

RAUS IAS TC 19 E 1007

19

cattle rearing. Their wealth was estimated in terms of their cattle. When they permanently settled in North India they began to practice agriculture. With the knowledge and use of iron they were able to clean forests and bring more lands under cultivation. Carpentry was another important profession and the availability of wood from the forests cleared made the profession profitable.

Q.94) Ans. (a) Explanation: •

The NBPW phase marked the beginning of the second urbanization in India.



It is also notable that the NBPW pottery was made of very fine material and apparently served as tableware for the rich.

Q.95) Ans. (d) Explanation: •

In the Indus Valley Civilization, there was a great progress in all spheres of economic activities, such as agriculture, industry, crafts and trade. Wheat and barley were the main crops grown, besides sesame, mustard and cotton.



Surplus grain was stored in granaries. Animals like sheep, goats and buffaloes were domesticated. The use of horse is not yet firmly established. A number of other animals were hunted for food, including deer.

Q.96) Ans. (a) Explanation: •



The cemeteries discovered around the cities like Mohenjodaro, Harappa, Kalibangan, Lothal and Rupar throw light on the burial practices of the Harappans. Complete burial and postcremation burial were popular at Mohenjodaro. At Lothal, the burial pit was lined with burnt bricks indicating the use of coffins. Wooden coffins were also found at Harappa. The practice of pot burials is found at Lothal, sometimes with pairs of skeletons. However, there is no clear evidence for the practice of Sati. The Harappan Script has not yet been deciphered.

Q.97) Ans. (c)

Q.98) Ans. (c) Explanation: •

The Sangha was governed on democratic lines and was empowered to enforce discipline among its members. Owing to the organised efforts made by the Sangha, Buddhism made rapid progress in North India, even during Buddha’s life time. Magadha, Kosala, Kausambi and several republican states of North India embraced this religion. About two hundred years after the death of Buddha, the famous Mauryan Emperor Asoka embraced Buddhism.

Q.99) Ans. (c) Explanation: •

Bimbisara belonged to the Haryanka dynasty. He consolidated his position by matrimonial alliances. His first matrimonial alliance was with the ruling family of Kosala. He married Kosaladevi, sister of Prasenajit.



He was given the Kasi region as dowry which yielded large revenue. Bimbisara married Chellana, a princess of the Licchavi family of Vaisali. This matrimonial alliance secured for him the safety of the northern frontier.



Bimbisara was a contemporary of both Vardhamana Mahavira and Gautama Buddha. However, both religions claim him as their supporter and devotee. He seems to have made numerous gifts to the Buddhist Sangha.

Explanation: •

The Rig Vedic Aryans were pastoral people and their main occupation was

RAUS IAS TC 19 E 1007

Q.100) Ans. (d) Explanation:

20



The immediate effect of Alexander’s invasion was that it encouraged political unification of north India under the Mauryas.



The system of small independent states came to an end.



Alexander’s invasion had also paved the way for direct contact between India and Greece. The routes opened by him and his naval explorations increased

RAUS IAS TC 19 E 1007

the existing facilities for trade between India and West Asia. However, his aim of annexing the north-western India to his empire was not fulfilled due his premature death. His authority in the Indus Valley was a short-lived one because of the expansion of the Mauryan Empire under Chandragupta Maurya.

21

Related Documents

Test #7
May 2020 5
Test 7
November 2019 7
Test 7
November 2019 4
Thematic Unit
June 2020 2
Ma Thematic
May 2020 7

More Documents from ""